Download as pdf or txt
Download as pdf or txt
You are on page 1of 74

418 Engineering Mathematics through Applications

6 C OMPLEX NUMBERS AND FUNCTIONS


aaaaa

6.1 INTRODUCTION
Complex numbers find applications in electric circuits, mechanical vibrating systems, etc. A
number of the form (a + ib), where a and b are real numbers and i = −1, is called a complex
number. In (a + ib), a is called the real part, written as R(a + ib) and b is called the imaginary
part, written as I(a + ib). A pair of complex numbers (a + ib) and (a – ib) are said to be
conjugate to each other. For complex number z(= a + ib), complex conjugate is denoted by
z(= a − ib).

Basic Properties of Complex Numbers


1. If z 1 = (a1 + ib1) and z2 = (a2 + ib2) are two complex numbers, then z1 = z2 implies a1 = a2
and b1 = b2, i.e. the numbers being equal means their real and imaginary parts are
separately equal.
Note: However, the inequalities between complex numbers such as: z1 > z2 or z1 < z2 has no meaning
since the field of complex numbers cannot be ordered.
2. Sum, difference, product and quotient of any two complex numbers is a complex
number.
(i) their sum z1 + z2 = (a1 + ib1) + (a2 + ib2) = (a1 + a2) + i(b1 + b2)
= (A + iB), a complex number
(ii) their difference z1 – z2 = (a1 + ib1) – (a2 + ib2) = (a1 – a2) + i(b1 – b2)
= (A – iB), a complex number
(iii) their product z1 · z2 = (a1 + ib1) (a2 + ib2) = (a1a2 – b1b2) + i(a1b2 + a2b1)
= C + iD, a complex number
z1 (a1 + ib1 ) (a1 + ib1 ) (a2 − ib2 )
(iv) their quotient = =
z2 (a2 + ib2 ) ( a2 + ib2 ) (a2 − ib2 )

a1a2 + b1b2 a b −ab


= + i 2 21 12 2 = C + iD,
a2 + b2
2 2
a2 + b2
418
Complex Numbers and Functions 419

3. Every complex number is expressible in the form r (cos θ + i sin θ) briefly written as
cis θ. For z = (a + ib) = r (cos θ + i sin θ), real and imaginary parts are

R(a + ib) = r cos θ, r2 = (a2 + b2 ),


 Y
I(a + ib) = r sin θ  θ = tan−1 
implying b
a
M(iy) P(z = x + iy)
The above form of the complex number is x
known as trigonometric form or polar form. Here
r = a2 + b2 (taking positive sign only is called y=b
r
the modulus or absolute value or norm of the
complex number z(= a + ib) denoted by |z| and θ
is called the argument or amplitude or phase of θ
z(= a + ib). X' O x=a L(x) X
Y'
Observations Fig. 6.1

(i) Although z and z have the same moduli, and equal value of the argument but different in sign.
(ii) As amplitude θ has an infinite number of values, the value θ which lies in between – π - π is called the
principal value of the argument. Unless otherwise stated, we take amplitude (z) to the mean principal
value.
(iii) Amplitude θ, if directed along the positive X-axis, is positive in the anti-clockwise direction and is
reckoned as negative in the clockwise direction.

6.2 (1) GEOMETRICAL REPRESENTATION OF IMAGINARY NUMBERS


Kuhn of Denzig was the 1st Mathematician who proposed the geometric representation of
imaginary number i.
Let OP be a real number represented by x along the positive direction of X-axis and OP'
by –x along OX'.
Since –x = i2x = i(ix) Y
It means that multiplication of real number x by i twice results o P"
in rotation of OP through two right angles to trace OP' which
naturally follows that the multiplication of real number by i is
o o
equivalent to the rotation of OP through one right angle to the X' P' O P X
position OP".
Here if Y'OY be a line perpendicular to the real axis X'OX,
then all the imaginary numbers are represented by points on
the Y'OY, called imaginary axis with positive imaginary numbers Y'
along OY and negative imaginary along OY'. Fig. 6.2

6.2 (2) GEOMETRICAL REPRESENTATIONS OF COMPLEX NUMBERS (ARGAND


DIAGRAM)
Geometrical representation of complex numbers came into being through the memories of
Jean Robert Argand, Paris 1806.
Consider the axis X'OX and Y'OY perpendicular to each other with all the positive real
numbers along OX and negative real axis along X'O.
420 Engineering Mathematics through Applications

As the multiplication of the real number by i is equivalent to the rotation of its direction
through 90°. Therefore, all the positive imaginary number along OY and negative imaginary
number along OY'. Let the point M on OY represent the imaginary number iy (Fig. 6.1)
Complete the rectangle OLPM so that the cartesian co-ordinate (x, y) uniquely represents
the complex number z = x + iy on the complex plane Z. This diagram which geometrically
represents the complex number Z is called Argand’s diagram.

Example 1: Express − 3 + i into modulus amplitude form and find its principle argument.

Solution: Given a + ib = − 3 + i with a = − 3, b = 1.


So that r = a2 + b2 = 3 + 1 = 2
− 3 1
Here, r cos θ = − 3 , i.e. cos θ = and r sin θ = 1, i.e. sin θ =
2 2
Here cosθ is negative and sin θ is positive, collectively implies θ should lie in the 2nd
 π 5π
quadrant with value θ =  π −  ± 2nπ = ± 2nπ; n = 0, 1, 2, …
 6 6  5π 
i ± 2nπ
So modulus amplitude form of − 3 + i is 2e  6  with the principle argument lying

between –π ≤ θ ≤ π and with value − .
6
Example 2: Show that the multiplication of a complex number by i results in a clockwise
rotation of the corresponding vector through right angle.

Solution: Let z = a + ib = r cisθ = re i θ Y


π
π i  θ+ 
So that  2
iz = i(reiθ) = e 2 (reiθ) = re iz
z
π
θ+ 2
 π π
Clearly the argument of iz is  θ +  which is more
2 2
than the argument of z.
Hence OZ is rotated through a right angle in θ
anticlockwise direction (Fig. 6.3) O X

Example 3: Reduce 1 + sin α + icos α in modulus


argument form. Fig. 6.3
 π − α ,
Solution: Here, r cos θ = 1 + sin α = 1 + cos
2 
 π − α
r sin θ = cos α = sin
2 
2 2
 π   π 
So that r = 1 + cos  − α  + sin  − α  
 2   2 
Complex Numbers and Functions 421

π π π
= 1 + cos2  − α + 2 cos  − α + sin2  − α
2  2  2 

 π   π α  π α
= 2 1 + cos  − α  = 2 2 cos2  −   = 2 cos  − 
 2    4 2   4 2

π π α π α
sin  − α  2 sin  −  cos  − 
tan θ = 2  = 4 2 4 2
and
 π
1 + cos  − α   2 π α
2 cos  − 
2  4 2

π α
or tan θ = tan  − 
 4 2

 π α  π α  π α 
Thus, (1 + sin α) + i cos α = r cis θ = 2 cos  −   cos  −  + i sin  −  
 4 2  4 2  4 2 

Example 4: Convert 12 ∠–60° to the rectangular form.

Solution: Take 12 ∠–60° = a + ib = r (cos θ + i sin θ)


1
So that r cos θ = 12 cos(− 60°) = 12 × =6
2
 3
r sin θ = 12 sin(− 60 °) = 12 ×  − = −6 3
 2 
and

Resulting in, r(cos θ + i sin θ) = a + ib = 12 ∠ − 60° = 6 − i 6 3


n
 1 + i =
Example 5: Find the smallest positive integer n for which 
 1 − i 
1

n n
 1 + i = 1  1 + i × 1 + i = 1
Solution:   ⇒  
1 − i 1 − i 1 + i
n
 1 − 1 + 2i  = 1
or  1+ 1  ⇒ in = 1 = (i)4 ⇒ n=4

a + ib 1 + iz
Example 6: If a2 + b2 + c2 = 1, b + ic = (1 + a)z, prove that =
1+c 1 − iz

b + ic
Solution: Given that b + ic = (1 + a)z or z=
1+ a
Now taking right hand side,
b + ic
1+ i
1 + iz
= 1 + a = (1 + a + ib) − c = (1 + a + ib) − c
1 − iz 1 − i b + ic (1 + a − ib) + c (1 + a + c) − ib
1+ a
422 Engineering Mathematics through Applications

=
[(1 + a + ib) − c ] × [(1 + a + c) + ib ]
[(1 + a + c) − ib ] [(1 + a + c) + ib ]
On rewriting above equation,
1 + iz [(1 + a + ib) − c ] [(1 + a + ib) + c ]
= ×
1 − iz [(1 + a + c) − ib ] [(1 + a + c) + ib ]

(1 + a2 − b2 − c2 + 2a) + 2ib(1 + a)
=
1 + (a2 + b2 + c2 ) + (2a + 2c + 2ac)

2(a2 + a + ib + iab)
= 2 2 2
2(1 + a + c + ac) , using a + b + c = 1
(1 + a)(a + ib) (a + ib)
= =
(1 + a)(1 + c) (1 + c)

ASSIGNMENT 1
1. Express the following in the form a + ib

2−i 3 1

1
(i) (ii) (2 + i)2 (2 − i)2
1+ i
2. Express (1 – cos α + sinα) in modulus amplitude form.
1 1
3. If + = 1; x, y, u, v being real quantities, express v in terms of x and y.
x + iy u + iy

iy 3y + 4i
4. If x and y are real numbers, solve the equation −
ix + 1 3x + y

5. Find what curve zz + (1 + i)z + (1 − i)z = 0 represent?


6. If z = 1 + i, find
1
(i) z2, (ii) ; and plot them on the Argand diagram.
z
7. In the Argand diagram, show that 9 + i, 4 + 3i, –8 + 8 i and –3 – 4 i form a square.
8. If |z1| = |z2| and amp (z1) + amp (z2) = 0, then show that z1 and z2 are complex conjugate
to each other.

6.2(3) SOME GEOMETRICAL RESULTS


(A) Geometric Representation of z1 + z2
Let P, Q represent the complex numbers z 1 = x1 + i y1 and z2 = x 2 + iy2. Complete the
parallelogram OPRQ. Draw PL, QM and RN ⊥ s to OX. Also draw PK ⊥ RN (See Fig. 6.4).
Since ON = OL + LN = OL + OM = x1 + x2, [³ LN = PK = OM]
Complex Numbers and Functions 423

NR = NK + KR = LP + MQ = y1 + y2 Y

The co-ordinates of R are (x1 + x2, y1 + y2) and it represents R


the complex number
z = (x1 + x2) + i(y1 + y2) = (x1 + iy1) + (x2 + iy2) = z1 + z2 Q
Thus, the point R which is extremity of the diagonal of z1+z2 z2
the parallelogram having OP and OQ as adjacent sides,
P
represents the sum of the complex numbers P(z1) and Q (z2) z2 K
such that |z1 + z2| = OR and amp (z1 + z2) = ∠ XOR z1

X
(B) Geometric Representation of z1 – z2. O M L N
Let P, Q represent the complex numbers z1 = x1 + iy1 and z2 = Fig. 6.4
x2 + iy2. Then the subtraction of z2 from z1 may be taken as Y
addition of z1 to – z2. Q z1–z2
Produce QO backwards to R such that OR = OQ. Then the co- P
ordinates of R are evidently (–x2, –y2) and so it corresponds to the
complex number (–x2 –iy2) = –z2. z2
z1
Complete the parallelogram ORSP, then the sum of z1 and
uuur uuur X'
O
X
–z2 is represented by OS, i.e. z1 − z2 = OS = QP . (See Fig. 6.5.) –z 2 S
Hence the complex number z1 – z 2 is represented by the
vector QP.
R
(C) Geometric Representation of z1z2
Let P, Q represent the complex numbers Y'
Fig. 6.5
z1 = x1 + iy1 = r1(cos θ 1 + i sin θ 1)
Y
and z2 = x2 + iy2 = r2(cos θ 2 + i sin θ 2) R
Measure OA = 1 unit along OX. Construct ∆ OQR on OQ
directly similar to ∆OAP, so that
OR OQ
= , i.e. OR = OP·OQ = r1r2 Q
2
r1 r

OP OA
r2
and ∠ AOR = ∠ AOQ + ∠ QOR = ∠ AOQ + ∠ AOP = θ 2 + θ 1
θ1 P
∴ P represents the number r1r2 [cos(θ 1 + θ 2) + i sin(θ 1 + θ 2)]. r1
(See Fig. 6.6.)
θ1 θ2
Hence the product of two complex numbers z1 and z2 is X
O A
represented by the point R, such that
Fig. 6.6
(i) |z1z2| = |z1| . |z2| (ii) amp(z1z2) = amp(z1) + amp(z2)

z1
(D) Geometric Representation of
z2
Let P, Q represent the complex numbers
z1 = x1 + i y1 = r1(cos θ 1 + i sin θ 1)
and z2 = x2 + i y2 = r2(cos θ 2 + i sin θ 2)
424 Engineering Mathematics through Applications

Measure OA = 1 unit and construct triangle OAR on OA Y


directly similar to the triangle OQP, so that P

OR OP OP r1
= , i.e. OR = =
OA OQ OQ r2
Q
∠ XOR = ∠ QOP = ∠ AOP – ∠ AOQ = θ 1 – θ 2.

2
and

r1 r
r2
∴ R represents the number
R
 r1  r 1/r 2
 r  [ cos(θ1 − θ2 ) + i sin(θ1 − θ2 )]. (See Fig. 6.7) θ1
2 θ2
X
Hence the complex number z1/z2 is represented by the O A
point R such that Fig. 6.7
z1 z z 
(i) = 1 (ii) amp  1  = amp(z1) − amp(z2 )
z2 z2  z2 

Example 7
(i) The modulus of the sum of two complex numbers is always less than or at the most
equal to their moduli.
or
If z1 and z2 be any two non–zero complex numbers, prove that
|z1 + z2| ≤ |z1| + |z2|,
(ii) The modulus of the difference of two complex numbers is greater or equal to the
difference of their moduli. Y
or
If z1 and z2 be two complex numbers, prove that R(z 1+z 2)
|z1 – z2| ≥ |z1| – |z2|
)
Q(Z 2
Geometrical Proof: (i)
Let P and Q represent the two complex numbers z1 and z2
P(Z1)
respectively in the Argand plane. Complete the
parallelogram OPRQ.
Here OP = |z1|, OQ = PR = |z2|, so that OR = |z1 + z2| O X
Now from the ∆OPR, OP ≤ OP + PQ or OC ≤ OP + OQ Fig. 6.8
∴ |z1 + z2| ≤ |z1| + |z2|
Alternately: Let z1 = r1(cos θ1 + i sin θ1), z2 = r2(cos θ2 + i sin θ2)
∴ |z1 + z2| = |r1(cos θ1 + i sin θ1) + r2(cos θ2 + i sin θ2)|
= |(r1 cos θ1 + r2 cos θ2) + i (r1 sin θ1 + r2 sin θ2)|
= (r1 cos θ1 + r2 cos θ2 )2 + (r1 sin θ1 + r2 sin θ2 )2

= r12 + r22 + 2r1r2 (cos θ1 cos θ2 + sin θ1 sin θ2 )

= r12 + r22 + 2r1r2 cos(θ1 − θ2 )


Complex Numbers and Functions 425

z1 + z2 ≤ r12 + r22 + 2r1r2 = (r1 + r2 ) = z1 + z2 [³ cos (θ1 – θ2) ≤ 1]


Cor. |z1 + z2 + … + zn| ≤ |z1| + |z2| + … + |zn|
7(ii) Left for readers

Example 8: If pair opposite vertices of a square be represented by the complex numbers


1 + 2i, 3 + 4i, show that the other pair is represented by 3 + 2i, 1 + 4i.

Solution: Take the square ABCD with vertices A and C as 1 + 2i and 3 + 4i, respectively.
Let the other two opposite vertices be (a + ib) and (c + id)
∴ ABr = OB
uuu uuur − OA
uuuur = (a + ib) − (1 + 2i) = (a − 1) + i(b − 2) … (1)
also uuur = OC
BC uuur − OB
uuur = (3 + 4i) − (a + ib) = (3 − a) + i(4 − b) … (2)
Now, whenever i is multiplied to a complex number, the number changes direction by
π/2, which implies that iAB
uuur = BC
uuur … (3) Y
or i [(a – 1) + i (b – 2)] = [(3 – a) + i (4 – b)] D(c, d) C(3, 4)
[(2 – b) + i (a – 1)] = [(3 – a) + i (4 – b)]
F
When two complex numbers are equal, their real and
imaginary parts are separately equal.

}
A(1, 2) B(a, b)
2 − b = 3 − a a + b = 5 a=3
⇒ ⇒ X' X
Thus, a − 1 = 4 − b  a − b = 1 b=2 O
Y' Fig. 6.9
And whence the vertex B is (3 + 2i)
Let the diagonals AC and BD intersects at F, then F is the common middle point of AC and
BD.
(1 + 2i) + (3 + 4i) (3 + 2i) + (c + id)
∴ = or (4 + 6 i) = (3 + c) + i (2 + d) … (5)
2 2


On equating real and imaginary parts
Vertex D represents (1 + 4 i).
3+c =4
2+d=6
⇒} c =1
d=4 } …(6)

Example 9: Find the locus of z given by


(i) |3z – 1| = |z – 3| (ii) |z| = |z – 2|
z+i z−i
(iii) is real (iv) is purely imaginary
z+2 z−2

Solution: Let z = (x + i y), so that


(z − 3) = (x − 3) + iy
(3z − 1) = (3x − 1) + 3iy } ... (1)

z−3 AP 3
Now, 3z − 1 = z − 3 ⇒ = =
1 BP 1 ... (2)
z−
3
Also, |z – 3| = |3z – 1| implies (x − 3)2 + y2 = (3 x − 1)2 + (3y)2
x2 + 9 – 6x + y2 = 9x2 + 1 – 6x + 9y2 or x2 + y2 = 1 ... (3)
426 Engineering Mathematics through Applications

Thus, from (2) and (3), we see that locus of P(z) is a circle with centre (0, 0) and radius 1
unit, and P moves in such a way that its distance from two fixed points, say, A(3, 0) and

B  , 0 , always bears a ratio 3. (Fig. 6.10)


1
3 
Y

P(z)

O B A
X
X1 (0, 0) 1,0 (3, 0)
3

Y1
Fig. 6.10

z−0 AP
(ii) Given |z| = |z – 2| ⇒ =1=
z−2 BP
In other words, P moves in such a way that its ratio of the distances from two points
A(0, 0) and B(2, 0) is always 1, i.e. it is the right bisector of AB.
(z + i)  x + iy + i  x + i(y + 1)
= =
(z + 2)  x + iy + 2  (x + 2) + iy
(iii) Here

x + i(y + 1) (x + 2) − iy 


=
(x + 2) + iy  (x + 2) − iy 

(x(x + 2) + y(y + 1)) + i((x + 2)(y + 1) − xy)


=
(x + 2)2 − i2 y2
x(x + 2) + y(y + 1) (x + 2)(y + 1) − xy
= +i
(x + 2)2 + y2 (x + 2)2 + y2
 z+i
Now  is real means its imaginary part is zero, i.e.
z + 2
(x + 2)(y + 1) – xy = 0 or x + 2y + 2 = 0
Whence the locus of z is a straight line.
 z − i  = x + i(y − 1)
(iv) Here
 z − 2  (x − 2) + iy

x + i(y − 1) (x − 2) − iy 


= ×
(x − 2) + iy  (x − 2) − iy 
Complex Numbers and Functions 427

[x(x − 2) + y(y − 1)] + i[(x − 2)(y − 1) − xy 


=
(x − 2)2 − i2 y2
x(x − 2) + y(y − 1) i(x − 2)(y − 1) − xy
= +
(x − 2)2 + y2 (x − 2)2 + y2
 z−i
Now,  is purely imaginary implies its real parts is zero.
z − 2
i.e. x (x – 2) + y (y – 1) = 0
x2 + y2 – 2x – y = 0
2
(x − 1)2 +  y −  =
1 5
 2  4

Clearly, the locus of z is a circle with centre  1,  and radius


1 5 units.
 2 2

Example 10: If the argument  z − 1  = π , prove that the point representing z on the
 z + 1 4
argand diagram lies on the fixed circle with its centre at the point i.

 z − 1 = π
Solution: Given  , i.e. the difference between arguments of (z – 1) and (z + 1) is
z + 1 4
π/4.
y 
θ1 = tan−1
}
,
We have z − 1 = (x − 1) + iy, so that x −1 
z + 1 = (x + 1) + iy y  …(1)
θ2 = tan−1 
x+1 
 z − 1 = θ − θ = π
Now given, arg 
z + 1
1 2 Y
4
y y π P (z )
tan−1 − tan−1 =
x−1 x+1 4 i
 y − y 
  π
tan−1  x − 1 x + 1  = O
y y X
1 +  4 ( O, O)
 x −1 x + 1
y [(x + 1) − (x − 1)] π
Fig. 6.11
= tan = 1
(x − 1)(x + 1) + y2 4
2y = (x2 – 1) + y2 or x2 + y2 – 2y – 1 = 0
(x – 0)2 + (y – 1)2 = 2
which represents a circle with centre (0, 1) and radius 2 units.
Hence the locus of z in the Argand diagram is the circle with its centre at the point i.
428 Engineering Mathematics through Applications

Example 11: If z1, z2 be the two non-zero complex numbers, show that
2 2
z1 + z2 + z1 − z2 = 2 z1 + z2 ( 2 2
)
Solution: Let OAuuuur = z1 , OB
uuur = z2 be the two given complex numbers and OC
uuur = z be another
number. Complete the parallelogram.
Now in ∆OAC by triangle law of forces,
uuuur + uuur
OA AC = OC uuur or OA uuuur + OB
uuur = OC
uuur or z1 + z2 = z ... (1)
uuur || uuur
(since in the parallelogram, OB AC )
In ∆OBA, uuur + BA
OB uuur = OA
uuuur i.e. uuur = OA
BA uuuur − OB
uuur
uuur = position vector of point A – position vector of point B = z1 – z2
i.e. BA ... (2)
Clearly in the parallelogram OACBO, OC ur and BA uuur are two C(z)
diagonals represented by complex numbers (z1 + z2) and (z1 – z2)
respectively.
Now in any parallelogram, the sum of the squares of the
diagonals is equal to twice the sum of the squares of the two
sides, i.e. A(z1)
B(z2) D
OC2 + BA2 = 2{OA2 + OB2}
or
2
uuur + BA
OC
2
uuur = 2 OA {
uuuur + OB
uuur
2 2
}
Mathematically, z1 + z2 + z1 − z2 = 2  z1 + z2 
2 2 2 2
O
Hence the proof. Fig. 6.12

Alternately: If z1 = r1(cos θ1 + i sin θ1) and z2 = r2 (cos θ2 + i sin θ2) then,


|z1 + z2|2 = |r1(cos θ1 + i sin θ1) + r2(cos θ2 + i sin θ2)|2
= |(r1cos θ1 + r2 cos θ2) + i (r1 sin θ1 + r2 sin θ2)|2
= (r1cos θ1 + r2 cos θ2)2 + (r1 sin θ1 + r2 sin θ2)2
= r12 + r22 + 2r1 r2cos θ1 cos θ2 + 2r1 r2 sin θ1 sinθ2
= r12 + r22 + 2r1 r2 cos (θ1 – θ2)
Similarly,
|z1 – z2|2 = |r1(cos θ1 + i sin θ2) – r2(cos θ2 + i sin θ2)|2
= |(r1cos θ1 – r2cos θ2) + i (r1sin θ1 – r2 sin θ2)|2
= (r1cos θ1 – r2 cos θ2)2 + (r1 sin θ1 – r2 sin θ2)2
= r21 + r22 – 2r1 r2cosθ1 cos θ2 – 2r1 r2sin θ1 sin θ2
= r12 + r22 – 2r1 r2 cos(θ1 – θ2)
On adding the two, we get
|z1 + z2|2 + |z1 – z2|2 = 2r12 + 2r22
2 2
= 2 z1 + 2 z2 = 2 z1 + z2 { 2 2
}
Hence the proof.
Complex Numbers and Functions 429

Example 12: If |z1 + z2| = |z1 – z2|, prove that the difference of amplitudes of z1 and z2 is π/2.

Solution: Let z1 = x1 + iy1 and z2 = x2 + iy2, so that


r1 = x12 + y12  r2 = x22 + y22 
 
 y   and  y 
θ1 = tan−1  1   θ2 = tan−1  2   ... (1)
 x1    x2  
Further, (z1 + z2) = (x1 + i y1) + (x2 + i y2) = (x1 + x2) + i (y1 + y2), ... (2)
(z1 – z2) = (x1 + i y1) – (x2 + i y2) = (x1 – x2) + i (y1 – y2) ... (3)
Also given that |z1 + z2| = |z1 – z2|

⇒ (x1 + x2 )2 + (y1 + y2 )2 = (x1 − x2 )2 + (y1 − y2 )2

⇒ x12 + x22 + 2x1 x2 + y21 + y22 + 2y1 y2 = x21 + x22 – 2x1 x2 + y21 + y22 – 2y1 y2
⇒ x1 x2 + y1 y2 = 0 ... (4)
which shows that the two vectors are perpendicular to each other.
Now, we need to prove that the difference of amplitude (the phase difference) between z1
and z2 is π/2.

Alternately: For z1 = r1cis θ1 and z2 = r2cis θ2

z1 + z2 = r12 + r22 + 2r1r2 cos(θ1 − θ2 ),


2

2  [See Example no. 7(i, ii) Page 424]


z1 − z2 = r12 + r22 + 2r1r2 cos(θ1 − θ2 ) 

Given, |z1 + z2|2 = |z1 – z2|2


⇒ 4r1 r2cos(θ1 – θ2) = 0
⇒ cos(θ1 – θ2) = 0 (Since r1 and r2 are not zero).
π
⇒ θ1 − θ2 =
2
Y
Example13: A rectangle is constructed in a complex
(z2)B A(1 3)
plane and its sides are parallel to the axes and its centre
is situated at the origin. If one of the vertices of the 60°

rectangle is 1 + i 3 , find the complex numbers 120°


60°
representing the other three vertices of the rectangle. X' X
O
Find also the area of the rectangle.

Solution: Let ABCD be the given rectangle with its sides 60°
parallel to the axis XOX/ and YOY/ with centre at the
origin O(0, 0) as shown in the Fig. 6.13. (z3 )C D(z4)
Here position of one of the edge, say, A(z1) is given
Y'
z1 = 1 + i 3 = r1(cos θ1 + i sin θ1) Fig. 6.13
430 Engineering Mathematics through Applications

⇒ r1 cos θ1 = 1, r1 sin θ1 = 3 so that r1 = 2 and θ1 = 60°

uuuur , i.e. z1 is such that it makes an angle of 60° with the initial axis and possesses
Hence OA
modulus of 2 units.
Now for the given rectangle, it is clear that each OA, OB, OC and OD are 2 units and OB uuur
uuuur through 60° further, and then OC
is obtained by rotating OA uuur is ordained by rotating OB
uuur
through 120° and then ODuuuur by rotating OC
uuur through 60° further.
θ1 = 60 ° 
θ2 = 60 ° + 60° = 120° = (π − 60°) 
and r1 = r2 = r3 = r4 = r = 2 units
θ3 = 120° + 120 ° = 240° = (π + 60°) 

θ4 = 240° + 60 ° = 300 ° = (2π − 60 °)
Therefore, z2 = r cis θ2 = 2[cos θ2 + i sin θ2]
= 2[cos(π – 60°) + i sin(π – 60°)]
 1 3
= 2 − + = −1 + 3
 2 2 
z3 = r cis θ3 = 2[cos(π + 60°) + i sin(π + 60°)]
 1 3
= 2 − − i = −1 − i 3
 2 2 
z4 = r cis θ4 = 2[cos(2π – 60°) + i sin(2 π – 60°)]
1 3
= 2 − i = 1− i 3
2 2 
Now, the area of the rectangle ABCD = BC × AB.
= (− 2)2 + 0 × 0 + (2 3)2 = 2 × 2 3 = 4 3

Example 14: An equilateral triangle constructed in the complex plane has its one vertex at
the point 1 + i 3 , find the complex numbers representing the other vertices. Find also
the area of the triangle. Y
A(1, 3)
Solution: Let ABC be the given triangle so that
∠ ABC = ∠ BCA = ∠ CAB = 60° or in other words if
O be the circumentre of the triangle,
B(– 2, 0)
∠ AOC = ∠ AOB = ∠ BOC = 120° 120° θ1 = 60
1 X
Hence clearly, first we need to locate the position X O
120°
of 1st edge or the angle made by OA uuuur with the
initial axis and then rotate OA uuuur through 120° to
obtain OBuuur (i.e. 2nd edge B) and further, through C(1, – 3)
Y1
uuur (i.e. 3rd edge C).
120° to obtain OC Fig. 6.14
Complex Numbers and Functions 431

Here given 1st edge A i.e. z1 = 1 + 1 3 = r cis θ1


So that r cos θ1 = 1 and r sin θ1 = 3 ⇒ r = 2 and θ1 = 60°
Clearly, θ2 = θ1 + 120° = 180°
θ3 = θ2 + 120° = 180° + 120° = 300°
Whence edge B, i.e. z2 = r cos θ2 = 2[cos π + i sin π] = –2
and the edge C, i.e. z3 = r cis θ3 = 2[cos300° + i sin300°]
= 2[cos(2 π – 60°) + i sin(2 π – 60°)]
1 3
= 2 − i = 1− i 3
2 2 
a+b+c
Now area of the ∆ = s(s − a)(s − b)(s − c), where s =
2

 1 1 1 
or ∆ = bc sin A = ca sin B = ab sin C
 2 2 2 

Here a = BC = 9 + 3 = 2 3 ,

b = CA = 0 + (2 2)2 = 2 3

c = AB = (− 3)2 + ( 3)2 = 2 3, otherwise also sides of equitatral are equal

2 3+2 3+2 3
∴ s= =3 3
2

{ } { } {
Whence, ∆ = 3 3 3 3 − 2 3 × 3 3 − 2 3 × 3 3 − 2 3 = 3 3 3 3 3 = 3 3 }
Example15: Show that the equation of the ellipse having foci at z1, z2 and major axis 2a, is
|z – z1| + |z – z2| = 2a. Also find its eccentricity. Further, find the locus given by
|z – 1| + |z + 1| = 3

Solution: Let P(z) be any point on the given ellipse with foci at S1(z1) and S2(z2)
So that S r = Position vector of P – Position vector of S1 or S1P = |z – z1|
1P
uuu ... (1)
Similarly S 2 P = Position vector of P – Position vector of S2 or S2P = |z – z2|
uuur ... (2)
We know that the sum of distances of the two foci from some general point on the ellipse
is equal to the major axis (see Fig. 6.15).
i.e. S1P + S2P = 2a or |z – z1| + |z – z2| = 2a ... (3)
which is the desired equation of the ellipse.
Also we know that S1S2 = 2ae, where e being the eccentricity.
z2 − z1
|OS2 – OS1| = 2ae, i.e. |z2 – z1| = 2ae implying e = …(4)
2a
432 Engineering Mathematics through Applications

P(z)

S1
A B
S2

O
Fig. 6.15

On comparing |z – 1| + |z + 1| = 3 with the equation (3), we see that locus of Z in this


equation is an ellipse with foci at z = ±1 and major axis 3.

Example 16:
(i) P1(z1), P2(z2) and P3(z3) be any three points then
 z − z2 
∠P1 P2 P3 = amp  3
 z1 − z2 

 z − z2 
(ii) Also prove that the points P1, P2, P3 are collinear, if  3 is wholly real.
 z1 − z2 
 z3 − z2 
(iii) Line P2P3 ⊥ P2P1, if 
 z1 − z2 
is wholly imaginary.
P3(z3)
P1(z1)
P2(z2)
Solution: Let P1(z1 ), P2 (z2) and P3 (z3) be the three
given points in the complex plane with reference point
O the origin as shown in the geometry.
Join OP 1 , OP 2 and OP 3 . Extend P 2 P1 and P2 P 3
θ2
backward meeting the axis XOX' in such a way that θ1
X1 X
they make angles θ1 and θ2 with it respectively. O

1 = z1 − z2 and P2 P3 = z3 – z2
Fig. 6.16
Now we have P 2 Pr
uuuu
∴ P1 P2 P3 = θ2 – θ1 = The difference for the external angle and the 2nd internal angle
= amplitude of P2 P3 – amplitude of P2 P1
= amplitude (z3 – z2) – amplitude (z1 – z2)

 z3 − z2 
= amplitude 
 z1 − z2 
(ii) If P1, P2, P3 are collinear then they all lie on a straight line or their angle with initial
axis is zero or π.
z3 − z2
= z (say) = r (cos θ + i sin θ) , (θ being 0° or π)
z1 − z2
Complex Numbers and Functions 433

r, for θ = 0
⇒ z= which is wholly a real number.
−r, for θ = π
(iii) if P2 P3 ⊥ P2 P1, then
 z3 − z2  π
∠PP
1 2 P3 = amp  = ,
 z1 − z2  2
z3 − z2 π
Say, = z, then clearly z = r cis = ri
z1 − z2 2
which is wholly imaginary

Example 17: If z1, z2, z3 be the three vertices of an isosceles triangle right-angled at z2,
prove that z12 + z32 + 2z22 = 2z2(z1 + z3).

Solution: Let ABC be the desired triangle with vertices A(z1), B(z2), C(z3) right-angled at
B(z2) as shown in Fig. 6.13.
The triangle being given an isosceles one, therefore, its two sides, say, BA = BC
i.e. |z1 – z2| = |z3 – z2|
Secondly it is right-angled at B(z2), i.e. ∠ABC = 90°
z1 − z2
Now, z = = r cis θ,
z3 − z2

z1 − z2 BA z −z 
where r= z = = = 1 and θ = arg .  1 2  = 90°
z3 − z2 BC  z3 − z2 

z1 − z2
Therefore, z= = r cis θ
z3 − z2

z1 − z2
= 1(cos 90° + i sin 90 °) = i
z3 − z2 C(z3 ) A (z 1 )

or (z1 – z2)2 = i2(z3 – z2)2


z12 + z22 – 2z1 z2 = – (z32 + z22 – 2z3 z2)
z12 + z32 + 2z22 = 2z2(z1 + z3)

ASSIGNMENT 2 B (z 2 )

1. The centre of a regular hexagon is at the origin Fig. 6.17

and one vertex is given by 3 + i on the Argand diagram. Determine the other vertices.
2. Find the locus of P(z), when
(i) |z – a| = k; (ii) amp(z – a) = α, where k and α are constants.
3. What domain of z-plane is represented by
(i) 2 ≤ |z + 3| < 4 (ii) I(z) > 2
434 Engineering Mathematics through Applications

π π
(iii) < amp(z) < (iv) |z + 2| + |z – 2| < 2
3 2
4. Find the locus given by |z – 1| + |z + 1| = 3
5. If z1, z2, z3 be the three vertices of an equilateral triangle, prove that
z12 + z22 + z32 = z1z2 + z2z3 + z3z1
6. What are the locii given by |z + 3| = k|z + 1| for k = 1 and 2.
7. Find the locus of the point z, where
z−a  z − a  = α, where k and α are constants.
=k (ii) amp 
 z − b 
(i)
z−b

6.3 DE MOIVRE’S THEOREM


Statement: If n be (i) an integer, positive or negative [PTU, 2005]
(cosθ + i sin θ)n = cos n θ + i sin nθ.
(ii) a fraction, positive or negative,
one of the values of (cos θ + i sin θ)n is (cos nθ + i sin nθ)
Proof: Case 1: when n is a positive integer.
By actual multiplication
cis θ1cis θ2 = (cos θ1 cos θ2 – sin θ1 sin θ2) + i (sin θ1 cos θ2 + cos θ1 sin θ2)
= cos (θ1 + θ2) + i sin(θ1 + θ2)
= cis (θ1 + θ2)
Similarly, cis θ1 cis θ2 cos θ3 = cos(θ1 + θ2 + θ3)
Proceeding in this way,
cis θ1.cis θ2 . cis θ3 …………… cis θn = cis (θ1 + θ2 + θ3 + ……………… + θn)
Now putting θ1 = θ2 = θ3 = ………………… = θn = θ, we obtain
(cis θ)n = cis nθ
Case 2: when n is a negative integer
Let n = –m, where m is a positive integer.
1 1
∴ (c i s θ)n = (cis θ)−m = = (By case I)
(cis θ)m cismθ

cos mθ − i sin mθ
=
(cos mθ + i sin mθ) × (cos mθ − i sin mθ)
[Multiplying the num. and deno. by (cos m θ – i sin mθ)]

cos mθ − i sin mθ
= = cos mθ − i sin mθ
cos2 mθ + sin2 mθ
= cos(–mθ) + i sin(–m θ) = cis(–m θ) = cis nθ [³ –m = n]
Complex Numbers and Functions 435

Case 3: when n is a fraction, positive or negative.


Let n = p/q, where q is a positive integer and p is any integer +ve or –ve.
q
 θ  θ
Now  cis q  =  cis q. q  = cis θ

∴ Taking qth root of both sides, cis(θ/q) is one of the q values of (cisθ)1/q, i.e. one of the

 θ
values of (cis θ)1/ q = cis   .
 q
p
 θ  p
Raise both sides to power p, then one of the values of (cis θ) p/ q
=  cis  = cis   θ
 q  q
i.e. one of the values of (cis θ)n = cis nθ. (By case 1 and 2)
Thus the theorem is completely established for all rational values of n.
Cor. 1. cis θ1.cis θ2.cis θ3 …………… cis θn = cis (θ1 + θ2 + θ3 + ………… + θn)
2. (cos θ – i sin θ)n = cos nθ – i sin nθ = (cos θ + i sin θ)–n
3. (cis mθ)n = cis mn θ = (cis nθ)m

[(cos5θ − i sin 5θ)2 (cos7θ − i sin 7θ)3 ]


Example 18: Prove that =1
[(cos 4θ − i sin 4θ)9 (cos θ − i sin θ)5 ]

Solution: Using DM Theorem,


(cos5 θ – i sin5 θ)2 = (cos 10 θ – i sin 10 θ) = (cos θ + i sinθ)–10
(cos 7 θ – i sin 7 θ)3 = (cos 21 θ – i sin21θ) = (cos θ + si nθ)–21
(cos4 θ – i sin 4θ)9 = (cos 36θ – i sin 36θ) = (cos θ + i sin θ)–36

(cos θ − i sin θ)−10(cos θ + i sin θ)−21 (cos θ + i sin θ)−31


Left Hand Side = = =1
(cos θ − i sin θ)−36 (cos θ + i sin θ)5 (cos θ + i sin θ)−31
(1 + i 3)13
Problem 19: Find the modulus and principal value of the argument of
( 3 − i)11
3 π
Solution: Say, 1 + i 3 = r1cisθ1 , where r1 = 2, θ1 = tan−1 = 60° =
1 3
1 π
Identically, 3 − i = r2cis θ2 , where r2 = 2, θ2 = − tan−1 = − 30 ° = −
3 6
13
π π
r113  cis  cis  13 ⋅ 
(1 + i √ 3)13  3 r113  3
= = 11 ⋅
( 3 − i)11  π 
11
r2 cis  11 ⋅ π 
r211  cis  −  
Now
  6  6

213 13π 11π  Because 1


=
1 
= cisθ
= cis cis , 
211
3 6  cis( − θ) (cis θ)−1

436 Engineering Mathematics through Applications

= 22 cis 
13 11
+ π
 3 6
π π π
= 22 cis  2π + + 2π −  = 2 cis
2
 3 6 6
π
Hence the modulus is 4 and principal argument
6
Example 20: Prove that the general value of θ which satisfy the equation
4π m
(cos θ + i sin θ)(cos θ + i sin θ)2 …… = 1 is where m is any integer.
n(n + 1)
Solution: (cos θ + i sin θ)(cos θ + i sin θ)2 … = (cosθ + i sin θ)1 + 2 + 3 + … + n
n(n + 1)
= (cos θ + i sin θ) 2

n(n + 1) n(n + 1) 
=  cos θ + i sin θ , …(1)
 2 2 
= 1 = (cos 2m π + i sin 2 mπ), Given …(2)
n(n + 1) 4mπ
∴ θ = 2mπ or θ =
2 n(n + 1)

Example 21: If (a1 + ib1) . (a2 + ib2) … … … (an + ibn) = A + iB


Prove that
− 1 b1 − 1 b2 − 1 bn −1 B
(i) tan a + tan a + … + tan a = tan A
1 2 n
(ii) (a12 + b12)(a22 + b22) + … + (an2 + bn2) = A2 + B2
b1
Solution: Let a1 = r1cosα1, b1 = r1sinα1 with α1 = tan−1 ;
a1
b2
a2 = r2cosα2, b2 = r2sinα2 with α2 = tan−1 ;
a2
.................. ..............................................
.................. ..............................................

bn
an = rncosαn, bn = rnsinαn with αn = tan−1 ;
an

A = rcosθ, B = rsinθ with θ = tan−1 b


a
(a1 + ib1) . (a2 + ib2) … (an + ibn) = A + iB
r1(cos α1 + i sin α1) r2(cos α2 + i sin α2) … rn(cos αn + i sin αn) = r(cos θ + i sin θ)
r1r2 … rn[cos (α1 + α2 + … + αn) + i sin(α1 + α2 + … + αn)] = r(cos θ + i sin θ)
Complex Numbers and Functions 437

Therefore, we see that modulus and amplitude are


r1r2 … rn = r or (a12 + b12)(a22 + b22) + … (an2 + bn2) = A2 + B2

b1 b b b
and tan−1 + tan−1 2 + …… + tan−1 n = tan−1
a1 a2 an a
Note: Alternately this problem can be taken under applications of log.

1 1
Example 22: Prove that (a + ib)n + (a − ib)n has n real roots and find those of
1 1
{1 + i 3 } 3 + {1 − i 3 } 3 .
 b
Solution: Let a = r cos θ, b = r sin θ so that r = a2 + b2 and θ = tan−1  
 a
1 1 1 1
∴ (a + ib)n + (a − ib)n = [r(cos θ + i sin θ)]n + [r(cos θ − i sin θ)]n

1
 1 1

= r n [cos(2πp + θ) + i sin(2πp + θ)]n + [cos(2πp + θ) − i sin(2πp + θ)]n 
 
1 
  2π p + θ   2π p + θ     2π p + θ   2π p + θ   
= r n  cos   + i sin    + cos   − i sin   
  n n   n n  

1
 2πp + θ 
= r n 2 cos 
 n 
1
= 2 { a2 + b2 }2n cos  2πp + tan−1 
1 b
n a

{ }
1
cos  2πp + tan−1 
1 b
=2 a2 + b2 n
n a
which is real and gives n real values corresponding to p = 0, 1, 2 ,…, n – 1
Now if a = 1, b = 3, n = 3, then
1 1 1 1
(a + ib)n + (a − ib)n = (1 + i 3)3 + (1 − i 3)3

π
1
= 2(1 + 3)2 × 3 cos  2pπ +  ;
1
p = 0, 1, 2
3 3
 6p π + π 
1
= 2(22 )2 × 3 cos  
 9 
438 Engineering Mathematics through Applications

 6pπ + π 
1
= 2.(2)3 cos  
 9 
π 7π 13π 
4 4 4
= 2 3 cos   , 2 3 cos   , 2 3 cos 
 9  9   9 

 πp 
4
= 2 3 cos   ; p = 1, 7, 13
 9

p+q
ϕ, so that
θ, q = cisϕ
Example 23: If p = cisθ = −i cot(θ − ϕ).
p−q
Solution: We have p = cos θ + i sin θ, q = cos ϕ + i sin ϕ
It implies p + q = (cos θ + cos ϕ) + i (sin θ + sin ϕ)
and p – q = (cos θ – cos ϕ) + i (sin θ – sin ϕ)

θ + ϕ θ − ϕ
cos θ + cos ϕ = 2 cos  cos 
 2   2 

θ + ϕ θ − ϕ
sin θ + sin ϕ = 2 sin  cos 
 2   2 

θ + ϕ ϕ − θ θ + ϕ θ − ϕ
cos θ − cos ϕ = 2 sin  sin  = −2 sin  sin 
 2   2   2   2 

θ + ϕ θ − ϕ
sin θ − sin ϕ = 2 cos  sin 
 2   2 
On sustituting these values,
θ + ϕ θ − ϕ θ + ϕ θ − ϕ
2 cos  cos  + 2 i sin  cos 
p+q  2   2   2   2 
=
p − q − 2 sin  θ + ϕ  sin  θ − ϕ  + 2 i cos  θ + ϕ  sin  θ − ϕ 
 2   2   2   2 

θ − ϕ  θ + ϕ θ + ϕ 
2 cos  cos  + i sin 
 2    2   2  
=
θ − ϕ  2 θ + ϕ θ + ϕ 
2 sin  i sin  + i cos 
 2    2   2  

θ − ϕ
cot 
 2  θ − ϕ
= = −i cot 
i  2 

Example 24: If a = cis α, b = cis β, c = cis γ, then


(b + c)(c + a)(a + b) β−γ γ −α α −β
= 8 cos cos cos
abc 2 2 2
Complex Numbers and Functions 439

Solution: Here a + b = (cos α + i sin α) + (cos β + i sin β)


= (cos α + cos β) + i (sin α + sin β)
α+β α −β α+β α −β
= 2 cos cos + 2i sin cos
2 2 2 2
α −β α+β α + β
= 2 cos cos + i sin
2  2 
… (1)
2
β−γ  β+γ β+γ
Similarly, b + c = 2 cos cos + i sin … (2)
2  2 2 
γ −α γ +α γ + α
c + a = 2 cos cos + i sin
2  2 
… (3)
2
Also, abc = cis α cis β cis γ = cos(α + β + γ) + i sin(α + β + γ) … (4)
On using (1), (2) and (3), we see that
α −β β−γ γ −α α+β α + β
(a + b)(b + c)(c + a) = 8 cos cos cos  cos + i sin ×
2 2 2  2 2 

 β+γ β+ γ  γ +α γ + α
cos + i sin × cos + i sin
 2 2   2 2 

α−β β−γ γ −α
(a + b)(b + c)(c + a) = 8 cos cos cos ×
2 2 2

  α + β β + γ γ + α  α + β β + γ γ + α 
 cos  2 + 2 + 2  + i sin  2 + 2 + 2  
 
α −β β−γ γ −α
(a + b)(b + c)(c + a) = 8 cos cos cos
2 2 2
(a + b)(b + c)(c + a) α −β β−γ γ −α
or = 8 cos cos cos (using (4))
abc 2 2 2
Hence the result.

Example 25: Simplify


α – cos β) + i (sin α – sin β)]n + [(cos α – cos β ) – i (sin α – sin β)]n
[(cosα

Solution: Let (cos α – cos β) = cos θ and (sin α – sin β) = sin θ … (1)
so that r2 = (cos α – cos β)2 + (sin α – sin β)2
= cos2 α + cos2 β – 2cos α cos β + sin2 α + sin2 β – 2 sin α sin β
= 2 – [cos (α + β) + cos (α – β)] + [cos(α + β) – cos (α – β)]
= 2 – 2cos (α – β) = 2[1 – cos (α – β)]
(α − β)
= 4 sin2
2
(α − β) n(α − β)
implying r = 2 sin and rn = 2n sin … (2)
2 2
440 Engineering Mathematics through Applications

Further,
1 1
2 cos (α + β) sin (α − β)
sin θ sin α − sin β α+β π α + β
tan θ = = = 2 2 = − cot = tan  + 
cos θ cos α − cos β − 2 sin 1 (α + β) sin 1 (α − β) 2 2 2 
2 2

π α +β
implying θ =  + . … (3)
2 2 
Now [(cos α – cos β) + i (sin α – sin β)]n + [(cos α – cos β) – i (sin α – sin β)]n
= [r (cos θ + i sin θ)]n + [r (cos θ – i sin θ)]n
= rn[(cos nθ + i sin nθ)] + rn[(cos nθ – i sin nθ)],
= 2rncos n θ, using (2) and (3)

Example 26: Prove that


n π θ nπ nθ 
(i) (1 + sin θ + i cos θ) + (1 + sin θ − i cos θ) = 2 cos 
n n n+ 1
− .cos  −
4 2  4 2
n
 1 + sin α + i cos α  = nπ nπ
(ii)  cos  − nα  + i sin  − nα 
 1 + sin α − i cos α   2   2 

Solution:
π π
(i) 1 + sin θ + i cos θ = 1 + cos  − θ + i sin  − θ
2  2 
π θ π θ π θ
= 2 cos2  −  + 2i sin  −  cos  − 
 4 2  4 2  4 2

π θ  π θ π θ 
= 2 cos  −   cos  −  + i sin  −   …(1)
 4 2   4 2  4 2 
Similarly,
π θ  π θ π θ 
1 + sin θ − i cos θ = 2 cos  −   cos  −  − i sin  −   …(2)
 4 2   4 2  4 2 
∴ (1 + sinθ + i cos θ)n + (1 + sin θ – i cos θ)n
π θ  nπ nθ  nπ nθ  
= 2n cosn  −   cos  − + i sin  −
 4 2   4 2   4 2  
π θ  nπ nθ  nπ nθ  
+2n cosn  −   cos  − − i sin  −
 4 2   4 2  4 2  
π θ nπ nθ 
= 2n cosn  −  .2 cos  −
 4 2  4 2
π θ nπ nθ 
= 2n+1 cosn  −  cos  −
 4 2  4 2
Complex Numbers and Functions 441

Identically, using (1) and (2), we have


n n
 1 + sin α + i cos α  =  1 + sin α + i cos α 
 1 + sin α − i cos α   1 + sin α − i cos α 

n
  π α  π α 
 cos  4 − 2  + i sin  4 − 2  
= 
 cos  π − α  − i sin  π − α  
  4 2  4 2  
n
  π α  π α  π α  π α 
 cos  4 − 2  + i sin  4 − 2  cos  4 − 2  + i sin  4 − 2  
= × 
 cos  π − α  − i sin  π − α  cos  π − α  + i sin  π − α  
  4 2  4 2  4 2  4 2  
n
  π α  π α 
2

 cos  −  + i sin  −   
= 
4 2 4 2  
 2 π α  2π α 
 cos  4 − 2  + sin  4 − 2  
 
n
 π α π α π α π α 
= cos2  −  + i2 sin2  −  + 2.i.cos  −  sin  −  
  4 2   4 2   4 2   4 2
n
 π α π α 
= cos 2  −  + i sin 2  −   ,
  4 2   4 2
(using cos 2θ – sin2θ cos 2θ, 2sinθ cosθ = sin 2θ)
nπ nπ
= cos  − nα + i sin  − nα .
 2   2 
Hence the proof.

Example 27: If cos α + cos β + cos γ = 0, then show that


(i) cos2 α + cos2 β + cos2 α = 0 or Σ cos 2α = 0
and sin2 α + sin2 β + sin2 α = 0 or Σ sin2 α = 0
α + β) + cos (β
(ii) cos (α β + γ) + cos (γγ + α) = 0 or Σcos (α
α + β) = 0
α + β) + sin(β
and sin(α β + γ) + sin(γγ + α) = 0 or Σ sin(α
α + β) = 0
3 3
(iii) cos α + cos β + cos γ =
2 2 2
or ∑ cos2 α =
2 2
3 3
and sin α + sin β + sin γ =
2 2 2
or ∑ sin2 α =
2 2
Solution: Let a = cis α, b = cis β, c = cis γ … (1)
Then a + b + c = cis α + cis β + cis γ
442 Engineering Mathematics through Applications

= (cos α + cos β + cos γ) + i (sin α + sin β + sin γ) = 0


(using the given conditions) ... (2)
Now for proof of (i), (ii) and (iii), we need to prove a–1 + b–1 + c–1 =0

By (1) a−1 = (cis α)−1 = cos α − i sin α



b−1 = (cis β)−1 = cos β − i sin β  …(3)
c−1 = (cis γ )−1 = cos γ − i sin γ 
On adding all, a–1 + b–1 + c–1 = (cos α + cos β + cos γ) – i (sin α + sin β + sin γ)
= 0 (using the given condition, a + b + c = 0) … (4)
1 1 1 bc + ca + ab
Now, 0= + + =
a b c abc
bc + ca + ab
0= i.e. bc + ca + ab = 0. …(5)
abc

bc = cis β × cis γ = cis(β + γ ), 



where ca = cis γ × cisα = cis(γ + α), …(6)
ab = cis α × cis β = cis(α + β) 
On substituting (6) in (5), we get
cis (α + β) + cis (β + γ) + cis (γ + α) = 0
i.e. cos (α + β) + cos (β + γ) + cos (γ + α) = 0 or ∑ cos(α + β) = 0
and sin (α + β) + sin (β + γ) + sin (γ + α) = 0 or ∑ sin(α + β) = 0
Now (a + b + c) = 0 (given)
∴ (a + b + c)2 = (a2 + b2 + c2) + 2(ab + bc + ca) = 0
a2 + b2 + c2 = 0 (on using (5)) …(8)
Now a2 = (cis α)2 = cis 2α, b2 = (cisβ)2 = cis2β, c2 = (cis γ)2 = cis2γ …(9)
On substituting (9) in (8), we get cis 2α + cis 2β + cis 2γ = 0
i.e. cos 2α + cos 2β + cos 2γ = 0 or ∑ cos 2α = 0

and sin 2α + sin 2β + sin2γ = 0 or ∑ sin 2α = 0


On rewriting cos 2 α = 2cos2 α – 1 in the above result, we get
(2cos2 α – 1) + (2cos2 β – 1) + (2cis2 γ – 1) = 0
3
or 2(cos2α + cos2β + cos2γ) = 3 or ∑ cos2 α =
2
Now in the above result, replacing cos2α by (1 – sin2α), we get
3
(1 − sin2 α) + (1 − sin2 β) + (1 − sin2 γ ) =
2
Complex Numbers and Functions 443

3 3 3
or sin2 α + sin2 β + sin2 β = 3 − = or ∑ sin2 α =
2 2 2
3
Hence ∑ cos2 α = = ∑ sin2 α
2
Example 28: If sin α + 2sin β + 3 sin γ = 0, cos α + 2cos β + 3 cos γ = 0, then prove that
sin3 α + 8 sin3 β + 27 sin3γγ = 18 sin(α
α + β + γ)
and cos3 α + 8cos3 β + 27cos 3 γ = 18 cos (α
α + β + γ)

Solution: Let a = cis α; b = 2 cis β; c = 3 cis γ … (1)


∴ a + b + c = cis α + 2 cis β + 3 cis γ
= (cos α + i sin α) + 2(cos β + i sin β) + 3(cos γ + i sin γ)
= (cosα + 2 cosβ + 3 cosγ) + i (sin α + 2 sin β + 3sin γ)
a + b + c = 0 as given (cos α + 2 cos β + 3 cos γ = 0 = sin α + 2 sin β + 3 sinγ )) …(2)
But we know that (a + b + c)2 = a3 + b3 + c3 – 3abc
∴ 3 3 3
a + b + c = 3abc (on using (2)) …(3)
where

a3 = (cis α)3 = (cos 3α + i sin 3α) 



b2 = (2 cisβ)3 = 8(cos 3β + i sin 3β)  …(4)
c3 = (3 cisγ )3 = 27(cos 3γ + i sin 3γ )
Further abc = cis α × 2cis β × 3 cis γ = 6 cis(α + β + γ) …(5)
On using (4) and (5) in (3), we get
(cos 3α + i sin 3α) + 8(cos 3β + i sin 3β) + 27(cos 3γ + i sin 3γ)
= 18{cos(α + β + γ) + i sin(α + β + γ)}
(cos 3α + 8cos 3β + 27cos 3γ) + i (sin 3α + 8 sin 3β + 27sin 3γ)
= 18{cos(α + β + γ) + i sin(α + β + γ)}
Comparing real and imaginary parts on both sides, we get
cos 3α + 8 cos 3β + 27cos 3γ = 18 cos(α + β + γ)
and sin 3α + 8sin 3β + 27sin 3γ = 18 sin(α + β + γ)
Hence the result.


Example 29: If α, β be the roots of x2 – 2x + 4 = 0, prove that α n + βn = cos .
3

Solution: The given equation is a quadratic in x. Hence its roots are given by
2 ± 4 − 4 × 4 2 ± −12 2 ± i 2 3
x= = =
2 2 2
π
Let α = 1 + i 3 = x1 + iy1 = r1 c i s θ1 ; r1 = 2, θ1 =
3
444 Engineering Mathematics through Applications

π
β = 1 − i 3 = x2 + iy2 = r2 c i s θ2 , ; r2 = 2, θ2 =
3
Now, αn + βn = (r cos θ + i r sin θ)n + (r cos θ – i r sin θ)n
= rn[cos n θ + i sin n θ + cosn θ – i sin nθ]
= rn[2cos n θ]

π nπ
= 2n. 2.cos n  = 2n+1 cos
 3 3

Example 30: If x2 – 2x cos θ + 1 = 0, show that x2n – 2xn cos nθ


θ + 1 = 0.

Solution: We know that x = −b ± b − 4ac


2

2a
−(− 2 cos θ) ± 4 cos2 θ − 4·1·1
=
2a
2 cos θ ± 4.(− 1)(1 − cos2 θ)
=
2

2 cos θ ± 2i sin θ
= = cos θ ± i sin θ
2
Hence the two of the roots are x1 = cos θ + i sin θ, x2 = cos θ – i sin θ
with x1 + x2 = 2cos θ and x1 x2 = (cos θ + i sin θ)(cos θ – i sin θ) = 1
Further, xn = cos nθ ± i sin n θ
Now the given equation is x2n – 2xn + 1 = 0

2 cos nθ ± 4 cos2 nθ − 4 2 cos nθ ± 2i sin nθ


The root, xn = = = cos nθ ± i sin nθ
2 2
Hence xn is the root of the given equation with sum of the root as x1 + x2 = 2 cos n θ and
product of the roots, x1 x2 = 1.

π
Example 31: If xr = cis , show that Lt x1 x2 …… xn = − 1
2r n→∞

 π  π
Proof: Here xr = cis r means for r = 1, x1 = cis
2   2

 π
r = 2, x2 = cis 2
2 

 π
r = n, xn = cis n
2 
Complex Numbers and Functions 445

π π π
Then x1.x2 .……… xn = cis .cis 2 ……… .cis n
2 2 2
π π π
= cis  + 2 + …… + n  (Using DM Theorem)
2 2 2 
1 π
The bracket part is a G.P. with common ratio, r = and the first term a = .
2 2
 a 
∴ x1.x2 . … .xn = cis 
 1 − r 

 π 
 
Lt x1.x2 ……xn = cis  2 
n →∞ 1
1 − 
 2

Lt x1.x2 ………xn = cis π = cos π + i sin π = −1


n→∞

Hence the solution.

ASSIGNMENT 3

(cos 3θ + i sin 3θ)4(cos 4θ − i sin 4θ)5


1. Prove that (i) =1
(cos 4θ + i sin 4θ)3 (cos 5θ + i sin 5θ)−4
4
 cos θ + i sin θ  = cos 8θ + i sin 8θ
(ii)  
sin θ + i cos θ 
(cos α + i sin α)4
(iii) = sin(4α + 5β) − i cos(4α + 5β)
(sin β + i cos β)5
2. If a = cis 2α, b = cis 2β, c = cis 2γ and δ = cis 2δ, prove that
ab c ab cd
(i) + = 2 cos(α + β − γ ) (ii) + = 2 cos(α + β − γ − δ)
c ab cd ab
3. Find the general value of θ which satisfies the equation
(cos θ + i sin θ)(cos 2θ + i sin 2θ) ……… (cos nθ + i sin nθ) = 1
n
 1 + sin α + i cos α  = cos  nπ − α + sin  nπ − α 
4. Prove that  n i n
 1 + sin a − i cos α   2   2 

1 1
5. If 2 cos θ = x + and 2 cos ϕ = y + , show that one of the value of
x y

1
(i) x y + xm yn is 2 cos(mθ + nϕ)
m n
[SVTU, 2007]
446 Engineering Mathematics through Applications

xm yn
(ii) + is 2 cos(mθ − nϕ)
yn xm
1
6. If 2 cos θ = x + prove that
x
x2n + 1 cos nθ
(i) 2 cos rθ = xr + 1 , (ii) 2n − 1
=
xr x + x cos(n − 1)θ
7. If sin α + sin β + sin γ = cos α + cos β + cos γ = 0, prove that
(i) sin2 α + sin2 β + sin2 γ = 0
(ii) sin3 α + sin3 β + sin3 γ = 3 sin(α + β + γ)
(iii) sin4 α + sin4 β + sin4 γ = 2 Σsin2(α + β)
(iv) sin(α + β) + sin(β + γ) + sin(γ + α) = 0

6.4 ROOTS OF A COMPLEX NUMBER


Statement: There are q and only q distinct values of (cos θ + i sin θ)1/q, q being an integer.
Since cos θ = cos(2n π + θ) and sin θ = sin(2n π + θ), where n is any integer.
∴ cis θ = cis(2n π + θ)
By De Moivre’s theorem, one of the values of
(2n + 1)π
(cis θ)1/ q = [cis(2nπ + θ)]1/ q = cis … (1)
q
Replacing n by the values 0, 1, 2, ..., (q – 1) successively, we get the following q values of
1
(cisθ)q ;
θ 
cis (for n = 0)
q 
(2π + θ) 
cis (for n = 1) 
q 

(4π + θ)
cis (for n = 2) 
q 
 …(2)
……………… …………

[2(q − 1)π + θ]
cis (for n = q − 1)
q 
Putting n = q in (1), we get a value of (cis θ) = cis(2 π + θ/q) = cis θ/q, which is the same as
1/q

the value for n = 0.


Similarly for n = q + 1, we get a value of (cis θ)1/q to be cis(2 π + θ/q), which is the same as
the value for n = 1 and so on.
Thus the value of (cis θ)1/q for n = q, q + 1, q + 2, etc. are the mere repetition of the q values
obtained in (2).
Moreover, the q values given by (2) are clearly distinct from each other, for no two of the
angles involved therein are equal or differ by a multiple of 2 π.
Hence, (cis θ)1/q has q and only q distinct values given by (2).
Complex Numbers and Functions 447

Example 32: Find the cube roots of unity and show that they form an equilateral triangle
in the Argand diagram.

Solution: If x be the cube root of unity, then


1 1 1 1
2nπ
x = (1)3 = (cos0 + i sin 0)3 = (cos 2nπ + i sin 2nπ)3 = (cis 2nπ)3 = cis ,
3
where n = 0, 1, 2.
For n = 0, x = cis 0 = 1
2π 2π 2π 1 3
n = 1, x = cis = cos + i sin =− +i
3 3 3 2 2
4π 4π 4π 1 3
n = 2, x = cis = cos + i sin =− −i
3 3 3 2 2
The three values of cube roots of unity are represented by the three points A, B, C in the
Argand diagram such that OA = OB = OC and angle AOB = 120°, angle AOC = 240°.
So these three points lie on a circle with centre O and unit radius such that angle AOB =
angle BOC = angle AOC = 120°. Hence A, B, C form an equilateral triangle.

Example 33: Find all the roots of x 12 – 1 = 0 and identify the roots which are also the roots
of x 4 – x 2 + 1 = 0.

Solution: Given x12 – 1 = 0 or x12 = 1 = cos 0 + i sin 0 = cos 2n π + i sin 2n π


1
x = [cos 2nπ + i sin 2nπ]12

2nπ 2nπ  nπ nπ
= cos + i sin = cos + i sin
 12 
12  6 6
… (1)

Putting n = 0, 1, 2, 3, 4, 5, 6, 7, 8, 9, 10, 11, we get all the 12 roots as below:


π π
n = 0, x1 = cos 0 + i sin 0; n = 1, x2 = cos + i sin ;
6 6
2π 2π 3π 3π
n = 2, x3 = cos + i sin ; n = 3, x4 = cos + i sin ;
6 6 6 6
4π 4π 5π 5π
n = 4, x5 = cos + i sin ; n = 5, x6 = cos + i sin ;
6 6 6 6
6π 6π 7π 7π
n = 6, x7 = cos + i sin ; n = 7, x8 = cos + i sin ;
6 6 6 6
8π 8π 9π 9π
n = 8, x9 = cos + i sin ; n = 9, x10 = cos + i sin ;
6 6 6 6
10π 10π 11π 11π
n = 10, x11 = cos + i sin ; n = 11, x12 = cos + i sin ; … (2)
6 6 6 6
By algebra, x12 = (x2 – 1)(x6 + 1)(x4 + x2 + 1).
Means, the roots of (x4 + x2 + 1) = 0 are the roots of x12 – 1 = 0, excluding the roots of
(x – 1) = 0 and (x6 + 1) = 0.
2

But the roots of (x2 – 1) = 0 are x2 = 1 or x = ± 1. … (3)


448 Engineering Mathematics through Applications

x = 1 implies x = cos 0 + i sin 0 and x = – 1 implies x = cos π + i sin π.


The roots of (x6 + 1) = 0 are
1 1 1
x = (−1)6 = (cos π + i sin π)6 = [cos(2nπ + π) + i sin(2nπ + π)]6
π π
x = cos(2n + 1) + i sin(2n + 1)  , n = 0, 1, 2, 3, 4, 5
 6 6
π π 3π 3π 5π 5π
viz. cos + i sin ; cos + i sin ; cos + i sin ;
6 6 6 6 6 6
7π 7π 9π 9π 11π 11π
cos + i sin ; cos + i sin ; cos + i sin ; … (4)
6 6 6 6 6 6
On excluding two roots of (3) and six roots of (4) from twelve roots of (2), we get the
desired four roots of the equation (x4 + x2 + 1) = 0 as:
π π 2π 2π 4π 4π 5π 5π
cos + i sin ; cos + i sin ; cos + i sin ; cos + i sin ;
3 3 3 3 3 3 3 3
Example 34: Prove that the nth root of unity form a geometrical progression. Also show
that the sum of these n roots is zero and their continued product is (–1)n–1
1
Solution: Let nth root of unity be x, i.e. x = 1n
 2r π  ,
1 1
x = (cis 0)n = (cis 2r π)n = cis 
or  n  r = 0, 1, 2, ……, n – 1 … (1)
Now roots for different values of r = 0, 1, 2, …, n – 1 from (1) are as follows:
 2π 
r = 0, x0 = cis  0 ×  = 1 ;
 n
1
 2π    2π  
r = 1, x1 = cis  1 ×  =  cis    = ρ (say) ;
 n    n 
2
2π   2π 
r = 2, x2 = cis  2 ×  =  cis  = ρ ;
2
 n  n
………………………………………………………………
………………………………………………………………
n −1
2π 2π
r = n–1, xn−1 = cis  ( n – 1)×  = cis   = ρn −1
 n   n 
Therefore, the n roots x0, x1, x2, …, xn–1 are ρ0, ρ1, ρ2, …, ρn–1 and their sum
Sn = ρ0 + ρ1 + ρ2 + … + ρn–1
ρ0 (1 − ρn )
=
1−ρ
  2π 2π  
n
 
1 − cos + i sin 
n n 
=
2π 2π
1 −  cos + i sin 
 n n
Complex Numbers and Functions 449

1−1
= =0
 2π 2π
1 − cos + i sin 
 n n
2π 2π
where 1 −  cos + i sin  ≠ 0 for all non zero n.
 n n
Hence the sum of n roots of unity is zero.
Further their continued product,
P = ρ0.ρ1.ρ2 ……… ρ n– 1 = ρ0 + 1 + 2+……+(n – 1)
0 + n −1 
n 
n(n −1)
 2 
=ρ =ρ 2

n −1
 2π
n /2

=  cis   
  n 
n −1
n 2π 
=  cis × (By DM Theorem)
 2 n 
= (cos π + i sin π)n – 1 = (–1)n – 1
Hence the continued product is (–1)n–1.

Example 35: Find the 7th root of unity and prove that the sum of their nth powers always
vanishes unless n be a multiple of 7, n being an integer, and then the sum is 7.

r
2π 
1 1 1

Solution: We know that [1]7 = (cos 2π r + i sin 2π r)7 = (cis 2π r)7 =  cis 
7
Putting r = 0, 1, 2, 3, 4, 5, 6, we get all the seven roots of unity and let these roots be

denoted by 1, ρ, ρ2, ρ3, ρ4, ρ5, ρ6, where ρ = cis
.
7
∴ Sum S of the nth powers of the seven roots = 1 + ρ n + ρ 2n + ρ 3n + ρ4n + ρ5n + ρ 6n
1 − ρ7n
= , (G.P. with common ratio ρ)
1 − ρn
In case when n is not a multiple of 7, ρ7n = (ρ7)n = (cis 2π)n = 1
i.e. 1 – ρ7n = 0 and 1 – ρn ≠ 0, as n is not a multiple of 7. Thus S = 0
In case when n is a multiple of 7, say n = 7p, then
S = 1 + (ρ7)p + (ρ7)2p + (ρ7)3p + (ρ7)4p + (ρ7)5p + (ρ7)6p = 1 + 1 + 1 + 1 + 1 + 1 + 1 = 7

2π 4π 6π
Example 36: Find the equation whose roots are 2 cos , 2 cos , 2 cos .
7 7 7
2πr
Solution: Let y = cos θ + i sin θ; θ= , r = 0, 1, 2,…… , 6 …(1)
7
Then y7 = (cos θ + i sin θ)7 ⇒ y7 = (cos 7θ + i sin 7θ)
Implying y7 = 1 or y7 – 1 = 0
450 Engineering Mathematics through Applications

⇒ (y – 1)(y6 + y5 + y4 + y3 + y2 + y + 1) = 0 … (2)
Leaving the factor (y – 1) which corresponds to θ = 0, we get
y6 + y5 + y4 + y3 + y2 + y + 1 = 0 … (3)
2π 4π 6π 8π 10π 12π
Its roots are y = cosθ, θ = , , , , ,
7 7 7 7 7 7
3
Divide (3) by y throughout, we get

 3 1  2 1  1
 y + y3  +  y + y2  +  y + y  + 1 = 0

 1
3
 1    1
2
  1
 y +  − 3  y +  +
  y +  − 2  + y +  + 1 = 0
 y  y   
 
y   y

 1
or x3 + x2 – 2x – 1 = 0, where x =  y +  = 2 cos θ …(4)
 y
Since
8π 6π 6π 
cos = cos  2π −  = cos , 
7  7  7

10π  4π  4π 
cos = cos 2π − = cos , are repeated roots.
7  7  7 
12π π π 
= cos  2π −  = cos
2 2
cos
7  7  7 
2π 4π 6π
Hence the roots of (4) are 2 cos , 2 cos , 2 cos .
7 7 7

2π 2π
Example 37: If a = cos + i sin , b = a + a2 + a4 and c = a3 + a5 + a6 , show that b and c
7 7
are the roots of the equation x2 + x + 2 = 0
2π 2π
Solution: Given a = cos + i sin
7 7
7
2π 2π
∴ a7 =  cos + i sin  = cos 2 π + i sin 2π = 1 …(1)
 7 7 
Now b + c = a + a2 + a3 + a4 + a5 + a6
= (1 + a + a2 + a3 + a4 + a5 + a6) – 1
1(1 − a7 )
= −1
1− a
(1 − 1)
= − 1 = −1 (³ a7 = 1) …(2)
1− a
Complex Numbers and Functions 451

and bc = (a + a2 + a4)(a3 + a5 + a6)


= a4 + a6 + a7 + a5 + a7 + a8 + a7 + a9 + a10
= a4 + a6 + 1 + a5 + 1 + a + 1 + a2 + a3 (³ a7 = 1)
= (1 + a + a2 + a3 + a4 + a5 + a6) + 2
=2 (³ 1 + a + a2 + a3 + a4 + a5 + a6 = 0) …(3)
∴ The equation whose roots are b and c is
x2 – (b + c)x + bc x = 0, i.e. x2 + x + 2 = 0.

ASSIGNMENT 4

( )
1
1. Mark the point on the Argand diagram, all the value of 1 + i√ 3 5 and verify that they
form a pentagon.
1 2
2. Find all the values of (i) (1 + i )4 (ii) ( − 1 + i )5
3. If ω is a complex cube root of unity, prove that 1 + ω + ω2 = 0.
4. Use DM theorem to solve the equation x4 – x3 + x2 – x + 1 = 0
 1

 Hint : (x − x + x − x + 1)(x + 1) = (x + 1), find x = (− 1)5 , except x = − 1
4 3 2 5

 
5. Solve the equation x6 + x5 + x4 + x3 + x2 + x + 1 = 0
 1

Hint : (x + x + x + x + x + x + 1)(x − 1) = (x − 1), find x = (1)7 , except x = 1
6 5 4 3 2 7

 
6. Find the roots common to the equations x4 + 1 = 0, x6 – i = 0.
π 3π 5π
7. Find the equation where roots are 2 cos , 2 cos , 2 cos .
7 7 7
8. Show that the roots of the equation (x – 1) n = (x) n, n being a positive integer are
1 rπ 
 1 + i cot  , r has the values 1, 2, 3, …, (n – 1).
2 n

6.5 (I) Expansion of sin n θ, cos nθ θ, tan nθθ in powers of sin θ, cosθ θ and tan θ respectively
(n being positive integer).
We have cos n θ + i sin n θ = (cos θ + i sin θ)n (by De Moivre’s theorem)
= cosn θ + nC1cos n–1 θ(i sin θ) + nC2 cosn–2 θ (i sin θ)2 + nC3 cosn–3θ(i sin θ)3 + ……
(by Binomial theorem)
= (cos θ – C2 cos θ sin θ + ……) + i ( C1 cos θ sin θ – C3 cos θ sin3 θ + …..)
n n n–2 2 n n–1 n n–3 …(1)
Equating real and imaginary parts from both sides, we get
cos n θ = cosn θ – nC2 cosn–2θ sin2 θ + nC4 cosn–4θ sin4θ – ….. …(2)
452 Engineering Mathematics through Applications

sin nθ = nC1 cosn–1θsin θ – nC3 cosn–3θ sin3 θ + nC5 cosn–5θsin5 θ – …… …(3)


Replacing every sin2 θ by (1 – cos2 θ) in (1) and every cos2 θ by (1 – sin2 θ) in (3), we get the
desired expansions of cos nθ and sin nθ.
Dividing (3) by (2),
n
C1 cosn −1 θ sin θ − n C3 cosn − 3 θ sin3 θ + nC5 cosn − 5 θ sin5 θ − ……
tan nθ =
cosn θ − n C2 cosn − 2 θ sin2 θ + n C4 cosn − 4 θ sin4 θ − ……
and dividing numerator and denominator by cosn θ, we get
n
C1 tan θ − n C3 tan3 θ + n C5 tan5 θ − ……
tan nθ = …(4)
1 − n C2 tan2 θ + nC4 tan 4 θ − ……

1 + cos7θ
Example 38: Prove that = (x3 − x2 − 2x + 1)2 , where x = 2 cos θ.
1 + cos θ
2
7θ  7θ
cos 
2 cos2  cos7φ 
2
1 + cos7θ 2  2  θ
Solution: L.H.S. = = =  , where 2 = φ … (1)
1 + cos θ 2 θ  θ   cos φ
2 cos  cos 
2  2
Now, cos7 φ = 7C0 cos7 φ – 7C2 cos5 φsin2 φ + 7C4 cos3 φ sin4 φ – 7C6 cos φ sin6 φ
= cos7 φ – 21 cos5 φ sin2 φ + 35 cos3 φ sin4 φ – 7 cos φ sin6 φ
= cosφ[cos6 φ – 21 cos4 φ sin2φ + 35 cos2 φ sin4 φ – 7 sin6 φ]
cos7φ
Thus = cos6 φ − 21cos4 φ sin2 φ + 35 cos2 φ sin4 φ − 7 sin6 φ …(2)
cos φ
 1 + x  = (1 + cos θ)
Given x = 2cos θ ⇒  2
 1 + x  = 2 cos2 θ = 2 cos2 φ
⇒  …(3)
2 2
 1 − x  = 2 cos2 θ = 2 sin2 φ
Similarly, …(4)
 2 2
On using (3) and (4), (2) becomes

 3 2 2
1− x 
3
1+  1+ x 1− x 1+ x
x x
cos 7 φ  
=  2  − 21  2  2  + 35  2  1 − 2  − 7  2 
cos φ   2   2   2 
   
 2 
   2   2  
 
  
 

1  x + 2   x + 2   x − 2  + 35  x + 2   x − 2  + 7  x − 2  
3 2 2 3
=  +
 2  
21
8  2   2   2   2  2 

1
= (x3 + 6x2 + 12x + 8) + 21(x3 + 2x2 − 4x − 8) + 35(x3 − 2x2 − 4x + 8)
64 
+7(x3 − 6 x2 + 12 x − 8)
Complex Numbers and Functions 453

1
=  64x3 − 64x2 − 128x + 64
64 
= x3 – x2 – 2x + 1
Therefore from (1),
2
1 + cos7θ  cos7φ  θ
= = (x3 − x2 − 2x + 1)2 , =φ
1 + cos θ  cos φ 
where
2

6.5 (II) Addition Formulae for any Number of Angles


We know cos(θ1 + θ2 + … + θn) + i sin(θ1 + θ2 + … + θn)
= (cos θ1 + i sin θ1)(cos θ2 + i sin θ2) … (cos θn + i sin θn) …(1)

Rewrite (cos θ1 + i sin θ1 ) = cos θ1(1 + i tan θ1), …(2)



(cos θ2 + i sin θ2 ) = cos θ2 (1 + i tan θ2 )
and so on 

∴ cos(θ1 + θ2 + …… + θn) + i sin(θ1 + θ2 + …… + θn)


= cos θ1 cos θ2 ………… cos θn (1 + i tan θ1)(1 + i tan θ2) …… (1 + i tan θn)
= cos θ1 cos θ2 ……… cos θn[1 + i (tan θ1 + tan θ2 + ……… + tan θn)
+ i2(tan θ1tan θ2 + tan θ2tan θ3 + ……)
+ i3(tan θ1tan θ2tan θ3 + ……)]
= cos θ1 cos θ2 ……… cos θn(1 + i s1 – s2 – is3 + s4 + ……) …(3)
where s1 = tan θ1 + tan θ2 + ……… + tan θn,
s2 = Σtan θ1tanθ2,
s3 = Σ tan θ1 tan θ2tan θ3, etc. …(4)
On equating real and imaginary parts in (3),
cos (θ1 + θ2 + ……… + θn) = cos θ1 cos θ2 ……… cos θn(1 – s2 + s4 – ……)
sin(θ1 + θ2 + ……… + θn) = cos θ1 cos θ2 ……… cos θn(s1 – s3 + s5 – ……)
Dividing the two,
s1 − s3 + s5 − ……
tan(θ1 + θ2 + …… + θn ) =
1 − s2 + s4 − ……

Example 39: If α, β, γ be the roots of equation x3 + px2 + qx + p = 0, prove that


tan–1α + tan–1 β + tan–1 γ = n π, except in one particular case.

Solution: Let α = tan θ1, β = tan θ2, γ = tan θ3 be the roots of the given equation.
a p
Then, S1 = ∑ tan θ1 = (−1)1 1 = − = −p = ∑ α
a0 1
a
S2 = ∑ tan θ1 tan θ2 = (−1)2 2 = q = ∑ αβ
a0
454 Engineering Mathematics through Applications

a3
S3 = ∑ tan θ1 tan θ2 tan θ3 = (−1)3 = −p = ∑ αβγ
a0
S1 − S3 (− p) − (− p)
∴ tan(θ1 + θ1 + θ3 ) = = = 0, except q ≠ 1
1 − S2 1−q

It implies tan(θ1 + θ2 + θ3 ) = tan nπ i.e. θ1 + θ2 + θ3 = n π


Hence, tan–1 α + tan–1 β + tan–1 γ = n π, except one particular value q ≠ 1.

π
Example 40: If tan–1 x + tan–1 y + tan–1 z = , then show that xy + yz + zx = 1.
2
(x + y + z) − xyz
Solution: We know that tan–1x + tan–1y + tan–1z = tan−1
1 − (xy + yz + zx)
π (x + y + z) − xyz
It implies that = tan−1
2 1 − (xy + yz + zx)

π (x + y + z) − xyz
tan =
2 1 − (xy + yz + zx)
π π
1 − (xy + yz + zx) sin 2 = cos 2 [(x + y + z) − xyz]
1 – (xy + yz + zx) = 0
i.e. xy + yz + zx = 1.

Example 41: Prove that the equation sin3 θ = a sin θ + b cos θ + c has six roots and that the
sum of six values of θ which satisfy it, is equal to an odd multiple of π radians.

Solution: We know that sin 3θ = 3 sinθ – 4sin3 θ


Then the given equation becomes,
3 sin θ – 4 sin3 θ = a sin θ + b cos θ + c … (1)
Now we need to prove that equation (1) has six roots and their sum is equal to an odd
multiple of π radians.
θ 
2 tan
2t 
2
Take sin θ = =
θ 1 + t2 
1 + tan2 
2 θ
θ  , where t = tan …(2)
1 − tan2
 2
2 1 − t2 
cos θ = =
θ 1 + t2 
1 + tan2
2 
On substituting values of sin θ and cos θ in (1), we get
3
 1 − t2 
3 
2t 
− 4 
2t 
= a 
2t 
   + b +c
1+ t 
2 1+ t 
2 1+ t 
2  1 + t2 
Complex Numbers and Functions 455

or (b – c)t6 – 2(a – 3)t5 + (b – 3c)t4 – 4(a + 5)t3 – (b + 3c)t2 – 2(a – 3)t – (b + c) = 0 …(3)
θ
which is a six degree equation in t  = tan  and has six roots.
 2
θ1 θ θ
Let its roots be t1, t2, t3, t4, t5, t6 or tan , tan 2 , …… , tan 6
2 2 2

−2(a − 3) 2(a − 3)  n an

Now S1 = ∑ t1 = (− 1)1 = Q Sn = (− 1) 
b−c b−c  a0 

(b − 3c) (b − 3c)
S2 = ∑ t1t2 = (−1)2 =
b−c b−c
−4(a + 5) 4(a + 5)
S3 = ∑ t1t2t3 = (− 1)3 =
b−c b−c
−(b + 3c) (b + 3c)
S4 = ∑ t1t2t3t4 = (−1)4 =−
b−c b−c
−2(a − 3) 2(a − 3)
S5 = ∑ t1t2t3 t4 t5 = (−1)5 =
b −c b −c
−(b + c) (b + c)
S6 = ∑ t1t2t3t4t5t6 = (−1)6 =−
b−c b−c

θ θ θ θ θ θ  S1 − S3 + S5
∴ tan  1 + 2 + 3 + 4 + 5 + 6  =
2 2 2 2 2 2  1 − S2 + S4 − S6

 θ + θ2 + θ3 + θ4 + θ5 + θ6  S1 − S3 + S5
⇒ tan  1
  = = ∞, (Q 1 − S2 + S4 − S6 = 0)
2 0
 θ + θ2 + θ3 + θ4 + θ5 + θ6   π
⇒ tan  1
  = tan  nπ + 
2 2
π
⇒ θ1 + θ2 + θ3 + θ4 + θ5 + θ6 = 2  nπ +  = (2n + 1)π
 2

6.5 (III) Expansion of sinm θ, cosm θ, sinm θ.cosm θ in a series of sines or cosines of
multiple of θ
1
If z = cos θ + i sin θ then = cos θ − i sin θ
z
By De. Moivre‘s theorem,
1
zp = cos pθ + i sin pθ and= cos pθ − i sin pθ
zp
1 1 1 1
∴ z + = 2 cos θ and z − = 2i sin θ; zp + p = 2 cos p θ and zp − p = 2i sin p θ
z z z z
456 Engineering Mathematics through Applications

These results are used in the expansion of sinθ, cosθ and their product in a series of sines
or cosines of multiple of θ.

Example 42: If sin 4 θ cos 3 θ = A 1 cos θ + A 3 cos3 θ + A 5 cos5 θ + A 7 cos 7θ


θ , prove that
A1 + 9A3 + 25A5 + 49A7 = 0.

Solution: Let x = cis θ, and x–1 = cis(–θ) ⇒  1


 x +  = 2 cos θ
x
 …(1)
 x − 1
 = 2i sin θ

 x
4 3
 1   1 
 x − x   x + x 
Now, sin θ cos θ = 
4 3
  
 2i   2 

=
1  4 −4 2 +6− 4 + 1  3 +3 + 3 + 1 
 x x  x x 
27 x2 x4   x x3 
1  7 1   5 1   3 1  1
=  x + 7  −  x + 5  − 3  x + 3  + 3  x +  
27   x x x x 
Using Result (1)
1
sin4 θ cos3 θ = [2 cos7θ − 2 cos 5θ − 6 cos 3θ + 6 cos θ]
27
1 1 3 3
⇒ sin4 θ cos3 θ = 6
cos7θ − 6 cos 5θ − 6 cos 3θ + 6 cos θ …(2)
2 2 2 2
Given sin4 θcos3 θ = A1 cos θ + A3 cos 3 θ + A5 cos 5 θ + A7 cos 7 θ, …(3)
On comparing (2) and (3), we get
1 1 3 3
A7 = , A5 = − 6 , A3 = − 6 , A1 = 6 …(4)
26 2 2 2

+ 9  − 6  + 25  − 6  + 49  6 
3 3 1 1
Now, A1 + 9A3 + 25A5 + 49A7 =
26  2   2   2 
1
= (3 − 27 − 25 + 49) = 0
26

ASSIGNMENT 5
sin 6θ
1. Express as a polynomial in cosθ.
sin θ
sin 7θ
2. Show that = 7 − 56 sin2 θ + 112 sin 4 θ − 64 sin6 θ.
sin θ
3. Show that 2(1 + cos 8θ) = (x4 – 4x2 + 2)2, where x = 2cos θ.
Complex Numbers and Functions 457

5t − 10t3 + t5
4. Show that tan 5θ = , where t = tan θ.
1 − 10t2 + 5t4
5. Show that 26 sin7 θ = 35 sin θ – 21sin 3 θ + 7 sin 5 θ – 7θ.
1
6. Show that cos7 θ = (cos7θ + 7 cos 5θ + 21cos 3θ + 35 cos θ. [Madras, 2003 ]
64
7. Show that sin8 θ = 2–7(cos 8 θ – 8 cos 6θ + 28 cos 4 θ – 56 cos 2 θ + 35) [Madras, 2001]
8. Show that 32 sin4 θ cos2 θ = cos 6θ – 2cos 4θ – cos 2θ + 2.
1
9. Show that sin5 θ cos2 θ = (sin 7θ − 3 sin 5θ + sin 3θ + 5 sin θ). [Madras, 2003]
64
10. Expand cos5 θ sin7 θ in a series of sine of multiples of θ. [Madras, 2001]
11. If cos5 θ = A cos θ + B cos 3θ + C cos 5θ, evaluate sin5 θ in terms of A, B, C.

6.6 COMPLEX FUNCTIONS


Definition: If for each value of the complex variable z(= x + iy) in a region R, we have one or
more values of ω(= u + iv), then ω said to be a complex function of z and we write
ω = u(x, y) + i v(x, y) = f(z) where u, v are real functions of x and y.
If to each value of z, there corresponds one and only one value of ω, then ω is said to be
1
single valued function of z otherwise a multi-valued function . For example, ω = is a single
z
valued function and ω = z is a multi valued function of z. The former is defined at all
points of z-plane except at z = 0 and the latter assumes two values for each value of z except
at z = 0.
Broadly we classify them under the following type:
(I) Exponential Function of Complex Variable
(II) Circular Functions of Complex Variable
(III) Hyperbolic Functions of Complex Variable
(IV) Inverse Hyperbolic Functions of Complex Variable
(V) Logarithmic Function of a Complex Variable

I Exponential Function of Complex Variable


1. Definition: When x is a real, we are already familiar with the exponential function
x x2 xn
ex = 1 + + +…+ + ………∞
1! 2! n!
In the same way, we define the exponential function of the complex variable z = x + iy,
as:
z z2 zn
ez or exp(z) = 1 + + + ……… + + ………∞
1! 2! n!
458 Engineering Mathematics through Applications

2. Properties
(i) Exponential form of z = reiθ
Substituting x = 0 in equation (1), we get
iy (iy)2 (iy)3 (iy)4
eiy = 1 + + + + + ………∞
1! 2! 3! 4!

 y2 y4   y3 y 5 
= 1 − + − ……… + i  y − + − ………
 2! 4!   3! 5! 
= cos y + i sin y
Thus, ez = ex e iy = ex(cos y + i sin y)
Also x + iy = r (cos θ + i sin θ) = re iθ. Thus, z = re iθ
(ii) ez is a periodic function having imaginary period 2πi, [ez+2n πi = ez e2nπi = ez]
(iii) ez is not zero for any value of z, since ez = ex+iy = re iθ or ex eiy = rei θ .

 As r = ex > 0, y = 0, eiy = 1
 
∴ e = e e = e ≠ 0.
z x iy x

II Circular Functions of Complex Variable


1. We know that ei θ = cos θ + i sin θ and e–iθ = cos θ – i sin θ
Solving these equations, the circular functions of real angles can be written as:
eiθ + eiθ eiθ − e−iθ
cos θ = , sin θ = and so on.
2 2i
Identically, we can write the circular functions of complex variable z by the equations:
eiz + e−iz eiz − e−iz sin z
cos z = , sin z = , tan z =
2 2i cos z
2. Properties:
(i) Circular functions are periodic: sin z, cos z are periodic functions having real period
2π while tan z have period π. Mathematically: sin(z + 2n π) = sin z, tan(z + n π) = tanz
etc.
(ii) cos z, sec z are even functions, while sin z, cosec z are odd functions.
Mathematically: cos(–z) = cos z and sin(–z) = –sinz
(iii) Zeros of sin z are given by z = ±2n π, zeros of cos z are given by
1
z = ± (2n + 1)π, n = 0, 1, 2, …
2
(iv) All the formulae for real circular functions are valid for complex circular functions
e.g. sin2 z + cos2 z = 1, sin(z1 ± z2) = sinz1 cosz2 ± cosz1 sinz2
3. Euler’s theorem eiz = cos z + i sin z.
Complex Numbers and Functions 459

III Hyperbolic Functions of Complex Variable

1. Definitions: Let z be a real or complex number


ez + e−z
(i) is defined as hyperbolic cosine of z and written as cosh z.
2
ez − e−z
(ii) is defined as hyperbolic sine of z and written as sinh z.
2
ez + e−z ez − e−z
Thus, cosh z = and sinh z =
2 2
sinh z ez − e−z 1 ez + e−z
Also define tanh z = = z ; coth z = =
cosh z e + e−z tanh z ez − e−z
1 2 1 2
sech z = = ; cosech z = =
cosh z ez + e−z sinh z ez − e−z
2. Properties:
(i) Periodic functions: sinhz and coshz are periodic functions having imaginary period
2πi.
Mathematically we can write: sinh (z + 2 πi) = sinh z; cosh (z + 2 πi) = coshz
(ii) Even and odd functions: cosh z is an even function while sinh z is an odd function
(iii) sinh 0 = 0, cosh 0 = 1, tanh 0 = 0
(iv) Relations between hyperbolic and circular functions:
eiθ − e−iθ eiθ + e−iθ
Since for all values of θ, sin θ = and cos θ =
2i 2
Substituting θ = ix, we have
e− x − ex ex − e− x ex − e− x ex − e− x
sin ix = =− = i2 = i. = i sinh x, (eiθ = ei.ix = e–x)
2i 2i 2i 2
e−x + ex
and cos ix = = cosh x
2
Thus, in brief we have:
(a) sin i x = i sinhx sinh i x = i sin x sinh–1x = –i sin–1(ix) 
(b) cos i x = cosh x cosh ix = cos x cosh–1x = i cos–1ix 
(c) tan ix = i tanhx tanh ix = i tan x tanh–1x = –i tan–1(ix) 

(d) cot ix = –i coth x coth ix = –i cot x coth–1x = –i cot–1(ix) 
(v) Some important formulae of hyperbolic functions:
1. cosh2x – sinh2x = 1
2. sech2x – tanh2x = 1
3. coth2x – cosech2x = 1
4. sin h(x ± y) = sinh x cosh y ± cosh x sinh y
5. cosh(x ± y) = cosh x cosh y ± sinh x sinh y
tanh x + tanh y
6. tanh(x ± y) =
1 ± tanh x tanh y
460 Engineering Mathematics through Applications

7. sinh 2x = 2 sinh x cosh x


8. cosh 2x = cosh2x + sinh2x = 2cosh2x – 1 = 1 + 2 sinh2x
2 tanh x
9. tanh 2x =
1 + tanh2 x
10. sinh 3x = 3sinhx + 4 sinh3 x
11. cosh 3x = 4 cosh3x – 3 cosh x
3 tanh x + tanh3 x
12. tanh 3x =
1 + 3 tanh2 x
x+y x−y
13. sinh x + sinh y = 2 sinh cosh
2 2
x+y x−y
14. sinh x − sinh y = 2 cosh sinh
2 2
x+y x−y
15. cosh x + cosh y = 2 cosh cosh
2 2
x+y x−y
16. cosh x − cosh y = 2 sinh sinh
2 2
Proof (1): For all values of θ, we know that cos θ + sin2 θ = 1
2

∴ Substituting θ = ix, we get cos2 ix + sin2 ix = 1 or cosh2 x – sinh2 x = 1


2 2
 ex + e−x   ex − e−x 
Also we can write cosh x − sinh x = 
2 2
 − 
 2   2 

1 2x
=
e + e−2 x + 2 − e2 x − e−2 x + 2  = 1
2
Identically, we can establish formulae (2) and (3).
1
Proof (4): sinh(x + y) = sin i(x + y) = −i [sin ix cos iy + cos ix sin iy ]
i
= – i [i sinh x cosh y + cosh x i sinh y] = sinh x cosh y + cosh x sinh y
ex − e− x ey + e− y ex + e− x ey − e− y
Also we can write, sinh x cosh y + cosh x sinh y = ⋅ + ⋅
2 2 2 2
ex + y − e−( x + y)
= = sinh(x + y)
2
Identically, we can establish the formulae (5) and (6).
3 tan A − tan3 A
Proof (12): tan 3A =
1 − 3 tan2 A
3 tan ix − tan3 ix
Substituting A = ix, tan 3ix =
1 − 3 tan2 ix
3(i tanh x) − (i tanh x)3 3 tanh x + tanh3 x
i tanh 3x = or tanh 3x =
1 − 3(i tanh x)2 1 + 3 tanh2 x
Complex Numbers and Functions 461

Identically, we can establish the formulae (7) to (11).


C+D C−D
Proof (16): cos C − cos D = −2 sin sin
2 2
x+y x−y
Substituting C = ix and D = iy, cos ix − cos iy = −2 sin i sin i
2 2

 x + y  x − y x+y x−y
or cosh x − cosh y = −2  i sinh   i sinh  = 2 sinh sinh
 2 2 2 2
Identically, we can establish the formulae (13) to (15).
x u
Example 43: If tan = tanh , prove that [KUK, 2008]
2 2
 π x
(i) tanx = sinhu and cos x cosh u = 1; (ii) u = loge tan  + 
4 2

x x
tan tan
u x
Solution: Given tanh = tan = 2 eu/2 − e−u/2 2
or =
2 2 1 eu/2 + e−u/2 1
By componendo and dividendo,

1 + tan x
(eu/2 + e−u/2 ) + (eu/2 − e−u/2 ) 2
=
(eu/2 + e−u/2 ) − (eu/2 − e−u/2 ) 1 − tan x
2

π x
= tan  + 
2eu/2
2e−u/2  4 2

 π + x
eu = tan
 4 2

 π + x .
Implying u = log tan This is the desired second part of the problem.
 4 2

u
= tan
x  π + x
Now tanh implies eu = tan … (1)
2 2  4 2

 π − x
e−u = tan … (2)
 4 2
From (1) and (2), we have
 π π x 
sin  +
x
sin  −  
eu − e−u 1   π x  π x 1  4 2 −  4 2
= tan + − tan − =
 4 2  4 2   2  
2 2  cos  π + x π x
cos  −  
 4 2  4 2  
462 Engineering Mathematics through Applications

  π x  π x  π x  π x 
 sin  4 + 2  cos  4 − 2  − sin  4 − 2  cos  4 + 2  
sinh u =  
π x π x
 2 cos  +  cos  −  
  4 2  4 2 

 
 sin A cos B − sin B cos A  sin(A − B)
= =
C+D C − D  cos C + cos D
 2 cos cos 
 2 2 

π x π x C+D π x C−D π x
wher A= + , B= − , = + , = −
4 2 4 2 2 4 2 2 4 2
π x π x
sin  + − + 
 4 2 4 2  sin x

sinh u = = = tan x
π cos x …(3)
cos + cos x
2
1
Now cosh2 u = 1 + sinh2 u = 1 + tan2 x = sec2 x = , (using sinh u = tan x)
cos2 x
or cosx cosh u = 1
Hence the complete solution of the part (i).
u x
Alternately: We have tanh = tan
2 2
 1 + tan x 
u x 1  2  = 1 log  tan  π + x  
= tanh tan = log 
−1
2 2 2 x 2   4 2  
 1 − tan 
 2
 π x 
u = log  tan  +  
  4 2
This is the second part of the question.
Now we have to prove cos x cosh u = 1
x u
1 − tan2 1 + tanh2
L.H.S. = 2⋅ 2
x u
1 + tan2 1 − tanh2
2 2
x u
But tan = = tanh , (given)
2 2
x x
1 − tan2 1 + tan2
So L.H.S. = 2⋅ 2 = 1 = R.H.S
x x
1 + tan2 1 − tan2
2 2
 π x 
if u = log  tan  +  
u x
Converse: tanh = tan [PTU, 2005]
2 2   4 2
Complex Numbers and Functions 463

 π x 
Given u = log  tan  +   implies eu = tan
 π + x
  4 2  4 2
u u 1 + tan x
e ⋅e =
2 2 2
x
1 − tan
2
1 + tan x
eu/2 2
=
e−u/2 1 − tan x
2
eu/2 − e−u/2 x
By componendo and dividendo, we have u/2 −u/2 = tan
e +e 2
u x
tanh = tan
2 2
 π θ  θ
Example 44: If cosh x = sec θ, prove that x = log  tan  −   and tanh2 = tan2 .
x
  
4 2  2 2
[NIT Kurukshetra, 2009]
−x
e +e
x
Solution: We know, cosh x = = sec θ implies e2x – 2 sec θex + 1 = 0
2
2 sec θ ± (4 sec2 θ − 4)
∴ ex =
2
1 + sin θ
= sec θ ± tan θ =
cos θ
θ θ θ θ
cos2 + sin2 + 2 cos sin
= 2 2 2 2
1 − 1 + cos θ
2
 cos θ + sin θ 
 2 2
=
 cos2 θ + sin2 θ  − 2 sin2 θ
 2 2 2
2
 cos θ + sin θ  θ θ
  cos + sin
= 2 2 = 2 2
θ θ θ θ
cos2 − sin2 cos − sin
2 2 2 2
θ
1 + tan
= 2 = tan  π − θ 
θ  4 2
1 − tan
2
 π θ 
Implying x = log  tan  −  
  4 2 
x θ
1 + tanh2 1 + tan2
Now given cosh x = sec θ implies 2 = 2 … (1)
2 x 2 θ
1 − tanh 1 − tan
2 2
464 Engineering Mathematics through Applications

 θ
1 + tan2 
x
1 + tanh2
 Since cosh x = 2 and sec θ = 1
= 2
 x cos θ 1 − tan2 θ 
 1 − tanh2
2 2
By componendo and divedendo, (1) becomes
 1 + tanh2 x  x  θ θ
− 1 − tanh2 1 + tan2  −  1 − tan2 
 2  
2 =  2   2
 1 + tanh2 x  x  θ θ
+ 1 − tanh2 1 + tan2  +  1 − tan2 
 2  2  2  2
x θ
i.e. tanh2 = tan2
2 2
IV Inverse Hyperbolic Functions of Complex Variable
1. Definition: Let sinh u = z, then u is called the hyperbolic sine inverse of z and is
written as u = sinh–1z. Similarly we define cosh–1z, tanh–1z.
The inverse hyperbolic functions like other inverse functions are many valued, but we
shall consider their principal values only.

2. Some Results sinh−1 z = log  z + ( z2 + 1)  …( i ) 

cosh−1 z = log  z + ( z2 − 1)  …( ii ) 
1 1+ z 
tanh−1 z = log …( iii )
2 1− z 

(i) Let sinh–1z = u, then z = sinh u = 1 ( eu − e− u )


2
1
2 z = eu − or e2 u − 2 z eu − 1 = 0
eu
2 z ± ( 4 z2 + 4)
This being a quadratic in eu, we get eu = = z ± ( z2 + 1)
2
∴ Taking the positive sign only, eu = z + (z2 + 1) or u = log  z + (z2 + 1) 
Similarly we can establish (ii).
eu − e−u
(iii) Let tanh–1z = u, then z = tanh u, i.e. z =
eu + e−u
1+ z eu
Applying componendo and dividendo, we get = −u = e2u
1− z e
1 + z
or 2u = log  , whence follows the result.
 1 − z

 π θ
Problem 45: Show that (i) sinh− 1 (tan θ) = log tan  +  ` [KUK, 2005]
4 2
θ
(ii) sec h− 1 (sin θ) = log cot (iii) tanh–1(cos θ) = cosh–1(cosec θ)
2
Complex Numbers and Functions 465

Solution: (i) sinh−1(tan θ) = log  tan θ + (tan2 θ + 1)  ,

(Using sinh −1
z =  z + (x2 + 1)  )
= log [tan θ ± sec θ]
sin θ 1 + sin θ 
= log 
1 
±  = log 
 cos θ 
, (taking + ve sign only) … (1)
 cos θ cos θ 

 sin2 θ + cos2 θ + 2 sin θ cos θ 


 2 2 2 2
= log  
 1 − 1 + cos θ

 2θ 2 θ θ θ
 sin 2 + cos 2 + 2 sin 2 cos 2 
= log  
  sin2 θ + cos2 θ  − (1 − cos θ) 
  2 2 

  sin θ + cos θ 
2

  
= log  2 2 
  sin2 θ + cos2 θ  − 2 sin2 θ 
  2 2 2 

 θ θ 
2
 sin + cos 
= log  2 2 
 cos2 θ − sin2 θ 
 2 2 

 sin θ + cos θ 
 2 2, θ
= log 
θ θ  (Taking cos 2 common)
 cos − sin 
 2 2
 1 + tan θ 
 2
= log 
θ
 1 − tan 
 2
 π θ 
= log  tan  +   . Hence proved.
  2 2 
Alternately: Let sinh–1(tanθ) = z implying sinh z = tanθ
ez − e−z
= tan θ or e2z − 2 tan θ ez − 1 = 0
2
2 tan θ ± 4 tan2 θ + 4
ez =
2
466 Engineering Mathematics through Applications

ez = tan θ ± sec θ
implying z = log[tan θ ± sec θ]
sin θ
= log 
1 
±
 cos θ cos θ 
1 + sin θ 
= log  , then onwards as above (1)
 cos θ 

sech−1 (sin θ) = cosh−1 


1  1
= cosh−1 , z = sin θ
(ii)  sin θ  z
1 
= log  +  2 − 1 
1
z  z  
 1 + (1 − z2 ) 
= log  
 z 
 1 + (1 − sin θ) 
2
= log  
 sin θ 
 1 + cos θ 
= log 
 sin θ 
 2 cos2 θ 
 2 
= log 
θ θ
 2 cos sin 
 2 2
θ
= log cot . Hence proved.
2
1 + z
log 
1
(iii) As we know that tanh−1 z =
2  1 − z
 2 cos2 θ 
+ θ
= log  2
1 1 cos 1
∴ tanh−1(cos θ) = log
2 1 − cos θ 2  2 θ
 2 sin 
2
1
 θ  2
2
 cos θ 
 cos   
= log  2  = log  2
 θ  θ 
 sin    sin 
 2    2

 cos θ 2 cos θ 
= log  2 × 2
 θ θ ,
 sin 2 cos 
2 2
Complex Numbers and Functions 467

1 + cos θ   2 cos2 θ = (1 + cos θ) and 2 sin θ cos θ = sin θ


= log  Q
 sin θ   2 2 2 

= log  + cot θ 
1
 sin θ 

(
= log cosec θ + cosec2θ − 1 )
= cosh–1(cosec θ), (Q log z + z2 − 1 = cosh−1 z )
V logarithmic Function of a Complex Variable
1. Definition: If z = x + iy and w = α + i β be related that ez = w, z is called the logarithm
of w to the base e and is written as z = log ew.
Now w = ez = ez · 1 = ez · w2nπi, (Since 1 = cos 2nπ + i sin 2nπ = ei2nπ)
Taking log on both sides, Log w = Log (e z + 2nπi ) = z + 2nπi, where n = 0, ±1, ±2, …
Hence the logarithm of complex number has an infinite number of values and is,
therefore, a many valued function. Log w (beginning with capital letter L) is called the
general value, log w is the principal value of a complex number (z, say). This principal
value is obtained by taking n = 0 in Log w.
2. We know that the logarithm of a negative quantity has no real value. But we can now
evaluate this.
e.g. loge(–3) = loge3(–1) = loge3 + loge(–1) = loge3 + iπ where –1 = cis π = ei π
3. Real and imaginary part of Log(x + i y):
log(x + i y) = 2inπ + log(x + iy)
= 2inπ + log [r(cos θ + i sin θ)]
−1  y 
where x = r cos θ, y = r sin θ so that r = (x2 + y2 ) and θ = tan  
x
∴ log(x + iy) = 2inπ + log(r e )iθ

  y
= 2inπ + log r + i θ = log (x2 + y2 ) + i  2nπ + tan− 1   
  x
z
4. If a = N, then z is called the logarithm of N to the base a and is written as z = logaN,
where N, a and z are complex numbers.
For a z = N taking logs on both sides, z loge a = log N implies ez loge a = elog N implies
ez⋅loge a = N . implies ez⋅loge a = az , where a and z both are complex numbers.
loge z
5. loga z =
loge a

Example 46: Find the real and imaginary parts of (α β)x +


α + iβ iy
468 Engineering Mathematics through Applications

β
Solution: Substitute α = r cos θ, β = rsin θ so that r = (α2 + β2 ) and θ = tan−1
α
Here (α + i β)x + iy = e(x + iy)Log(α + i β), (using az = ez⋅loge α )
= e(x + iy)[2inπ + log(α + iβ)
= e(x + iy)2inπ+ re 

= e(x + iy)[log r + i(2nπ + θ)]


= eA + iB = eA(cosB + i sinB)
where A = x logr – y(2nπ + θ) and B = y log r + x(2nπ + θ)
∴ Required real part = eA cos B and the imaginary part = eA sin B.
1 2 cos  π log2 
Example 47: Prove that ilog(1+ i) is e− 8 π 
4 

 1 log 2 + i tan−1 1⋅ log 1+ i tan−1 ∞ 


  
Solution: L.H.S. = i log(1+ i) = e log(1+ i)⋅log i = e  2
 1 log 2 + i π ⋅ i + π   π2 π log 2 
− 8 +i 4 
= e  2 4   2  = e 

π2 π log 2 π2
− i −  π log 2 π log 2 
=e 8e 4 =e 8
cos 4 + i sin 4 

π 2 log 2
∴ Required real part = eA cos B and the imaginary part= eA sin B, where A = − and B = π
8 4

i ··· ∞
Example 48: If ii = A + iB principal values only being considered, prove that
πA B
(i) tan = (ii) A2 + B2 = e–Bππ [NIT Kurukshetra, 2007]
2 A

Solution: Given iii…∞ = A + iB means iA + iB = A + iB (since…. iii…∞ = A + iB )


Taking logs on both sides, (A + iB) log i = log(A + iB)

(A + iB)  log 1 + i tan−1 ∞  = log(A2 + B2 ) + i tan−1


1 1 B
2  2 A

π
(A + iB) i  = log(A2 + B2 ) + i tan−1
1 B
 2 2 A
Now equating real and imaginary parts,
1 −Bπ
log(A2 + B2 ) = implying A2 + B2 = e–Bπ
2 2
B π B πA
and tan− 1 =A implying = tan
A 2 A 2
Complex Numbers and Functions 469

b
2 tan− 1
y a
Example 49: If (a + ib)p = mx + iy, prove that =
x log(a2 + b2 )

Solution: Taking logs on both sides of (a + ib)p = mx + iy, we get


p log(a + ib) = (x + iy)logm

1 b
implying p  log(a2 + b2 ) + i tan−1  = (x + iy)log m
2 a
Now equating real and imaginary parts,
1
p log(a2 + b2 ) = x log m …(i)
2
b
and p tan−1 = y log m ..(ii)
a

b b
y p tan −1 2 tan−1
Dividing the two, = a = a
x p log(a2 + b2 ) log(a2 + b2 )
2
π
− (4nπ + 1)
Example 50: Prove that ii = e 2 and show that its values form a geometrical
progression.

 iπ 

i ⋅ 2nπ i + log i  i ⋅ 2nπ i + log e 2 


i ⋅ log i
Solution: By definition, i = e i
=e = e  


i ⋅ 2nπ i +  −(4nπ+ 1)
π
=e  2 =e 2

π π π
− −9
On putting n = 0, 1, 2,…, we get the values of ii as e 2 ⋅ e−5 2 , e 2 , …, which form a G.P.
whose common ratio is e–2π.

Example 51: If (1 + i)(1 + 2i)…(1 + in) = (x + iy) , then show that 2.5.10 … (1 + n2) = (x2 + y2)

Solution: Given (1 + i)(1 + 2i) … (1 + in) = (x + iy),


Taking log on both sides,
log(1 + i)(1 + 2i)…(1 + in) = log(x + iy)
 1 log(12 + 12 ) + i tan−1 1  1 log(12 + 22 ) + i tan−1 2 …  1 log(12 + n2 ) + i tan−1 n
Implies
 2   2   2 
1 y
log(x2 + y2 ) + i tan−1
=
2 x
On equating the real and imaginary parts,
470 Engineering Mathematics through Applications

 1 log(12 + 12 ) +  1 log(12 + 22 ) + … +  1 log(12 + n2 ) = 1 log(x2 + y2 )


 2   2   2  2

1 1
log 2 ⋅ log 5 ⋅ log 10 …log(1 + n2 ) = log(x2 + y2 )
2
Implying
2
or 2 · 5 · 10 ·…… (1 + n2) = (x2 + y2)

  a − ib   = 2ab .
Example 52: Prove that tan  i log   [KUK, 2006, 07]
  a + ib   a2 − b2

b 2
Solution: Let a = r cosθ and b = r sin θ then tan θ = , r = a2 + b2 … (1)
a
 a − ib  
Now, L.H.S. = tan  i log  
  a + ib  
= tan i [log(a – ib) – log(a + ib)]
= tan i [log(re–i θ) – log(rei θ)]
= tan i [log r – i θ) – (logr + i θ)]
= tan i (–2i θ) = tan(–i22θ) = tan 2θ
b
2
2 tan θ a = 2ab (Using equation (1))
= =
1 − tan2 θ 1 − b2 a2 − b2
a2
2a
Example 53: If log(x + iy) = a + ib where a2 + b2 ≠ 0, show that tan log( x2 + y2 ) =
1 − a2 − b2
Solution: Given tan log(x + iy) = a + ib implies log(x + iy) = tan–1(a + ib)
y
Now log(x + iy) = logr cisθ = tan–1(a + ib), where r = x2 + y2 and θ = tan−1
x
1 y
or log(x2 + y2 ) + i tan− 1 = tan−1(a + ib) …(1)
2 x
1 y
Identically, log(x2 + y2 ) − i tan−1 = tan−1(a − ib) …(2)
2 x
Adding the two, log(x2 + y2) = tan–1(a + ib) + tan–1(a – ib)
(a + ib) + (a − ib)
= tan−1
1 − (a + ib)(a − ib)
2a
= tan−1
1 − a2 − b2
2a
or tan log(x2 + y2 ) =
1 − a2 − b2
Complex Numbers and Functions 471

Example 54: (i) tanhz + 2 = 0 (ii) cosz = 2

ez − e−z
Solution: (i) tanh z + 2 = 0 ⇒ = −2
ez + e−z
Apply Componendo and dividendo,
(ez + e−z ) + (ez − e−z ) 1 + (−2) − 1
= =
(ez + e−z ) − (ez − e−z ) 1 − (−2) 3

ez 1 i
=− or ez = , (taking positive sign only)
e−z 3 3
i
i.e. z = log = logr + eiθ,
3

π
, θ = tan −1 = tan−1 ∞ =  nπ + 
1 1 y
For x = 0, y = ;r=
3 3 x  2
1
+ i n +  π
− 1
z = log 3 2
 2

1  1
z = − log 3 + i n + π
2  2

ASSIGNMENT 6
1. Find the general value of (i) log(6 + 8i) (ii) log(–1) (iii) ii [MDU, 2005]
2. Show that (i) log (1 + itan α) = log (sec α)+ iα, where α is an acute angle.
3 − i = 2i  nπ − tan−1 1 
(ii) loge  
3+i  3
3. Find the modulus and argument (i) (1 + i)1 – i (ii) log (1 + i)
4. If (a1 + ib1)(a2 + ib2) …… (an + ibn) = A + iB, prove that

(i) ( a12 + b12 )( a22 + b22 )……( an2 + bn2 ) = A2 + B2


b1 b b B
(ii) tan
−1
+ tan−1 2 + … + tan−1 n = tan−1
a1 a2 an A

 a + ib  
5. Prove that log  a + ib  = 2 tan−1 b , hence evaluate cos  i log   .
 a − ib  a   a − ib  
x2 y2
6. If sin–1(x + iy) = log(A + iB), show that − = 1, where A2 + B2 = e2u.
sin2 u cos2 u
[KUK, 2007]
472 Engineering Mathematics through Applications

7. If i(α + iβ) = α + iβ, prove that a2 + b2 = e–(4n + 1)πβ [KUK, 2005]

(1 + i)x + iy β πx
8. If = α + iβ, prove that one of the value of tan−1 = + y log 2
(1 − i)x − iy α 2
9. Find all the roots of (i) sin Z = cosh 4 (ii) sinh Z = i.

6.7 REAL AND IMAGINARY PART OF CIRCULAR AND HYPERBOLIC FUNCTIONS:


(1) To separate real and imaginary parts of circular function:
(i) sin(x + iy); (ii) cos(x + iy); (iii) tan(x + iy);
(iv) cot(x + iy); (v) sec(x + iy); (vi) cosec(x + iy)

Proofs:
(i) sin(x + iy) = sinx cos iy + cosx sin iy = sin x cosh y + i cosx sinh y
Likewise, cos(x + iy) = cos x cosh y – i sin x sinh y
(iii) Let α + iβ = tan(x + iy) then α – iβ = tan(x – iy)
Adding 2α = tan(x + iy) + tan(x – iy)
sin(x + iy + x − iy) sin 2x sin 2x
i.e α= = = .
2 cos(x + iy)cos(x − iy) cos 2x + cos 2iy cos 2x + cosh 2y
Subtracting 2iβ = tan(x + iy) + tan(x – iy)
Likewise,
sin 2iy i sinh 2y
iβ = =
2 cos(x + iy)cos(x − iy) cos 2x + cosh 2y
sinh 2y
∴ β=
cos 2x + cosh 2y
sin 2x − i sinh 2y
Likewise, cot(x + iy) =
cosh 2y − cos 2x
(v) Let α + iβ = sec(x + iy) = sec(x + iy) then α – iβ = sec(x – iy)
Adding 2α = sec(x + iy) + sec(x – iy)
cos(x − iy) + cos(x + iy ) 2 cos x cos iy 2 cos x cosh y
i.e. α= = =
2 cos(x + iy)cos(x − iy) cos 2x + cos 2iy cos 2x + cosh 2y
subtracting, 2iβ = sec(x + iy) – sec(x – iy)

cos(x − iy) − cos(x + iy) 2 sin x sin iy 2i sin x sinh y


iβ = = =
2 cos(x + iy)cos(x − iy) cos 2x + cos 2iy cos 2x + cosh 2y

2 sin x sinh y
∴ β=
cos 2x + cosh 2y

sin x cosh y − i cos x sinh y


Likewise, cos ec(x + iy) = 2
cosh 2y − cos 2x
Complex Numbers and Functions 473

(2) To separate real and imaginary parts of hyperbolic functions:


(i) sinh(x + iy); (ii) cosh(x + iy); (iii) tanh(x + iy)
Proof:
(i) sinh(x + iy) = (1/i) sin i (x + iy) = (1/i) sin(ix – y)
= (1/i)[sin ix cos y – cos ix sin y]
= (1/i)[i sinhx cos y – cosh x sin y]
= sinh x cos y + i cosh x sin y
Likewise, cosh(x + iy) = cosh x cos y
(iv) If α + iβ = tanh(x + iy) = (1/i)tan(ix – y),
Then α – iβ = tanh (x – iy) = (1/i)tan (ix + y)
Adding 2α = (1/i) [tan (ix – y) + tan (ix + y)]
sin (ix − y + ix + y) (1/ i) sin 2ix sinh 2x
α= = =
i ⋅ 2 cos (ix − y) cos (ix + y) cos 2ix + cos 2y cos 2x + cos 2 y
Subtracting, 2iβ = (1/i)tan(ix – y) – tan(ix + y)

sin (ix + y) − (ix − y)


iβ = −
i ⋅ 2 cos(ix + y)cos(ix − y)

sin 2y sin 2y
β= =
cos 2ix + cos 2y cosh 2x + cos 2y

β) = x + iy, prove that


α + iβ
Example 55: If sin(α
x2 y2 x2 y2
(a) + =1 (b) − =1
cosh β sinh2 β
2
sin2 α cos2 α

Solution: (a) x + iy = sin(α + iβ) = sin α cosh β + i cos α sinh β


Separating real and imaginary parts,
x = sin α cosh β; y = cos α sinh β

x y
Rewrite as: sin α = and cos α =
cosh β sinh β

x2 y2
Squaring and adding + = 1 Hence proved the first part of the question.
cosh β sinh2 β
2

x y
(b) Again cosh β = and sinh β =
sin α cos α
Squaring and subtracting, we get
x2 y2
1= − , because cosh2β – sinh2β = 1. Hence proved.
sin2 α cos2 α
474 Engineering Mathematics through Applications

x2 y2 x2 y2
Example 56: If cosh(u + iv) = x + iy, prove that + = 1 and − =1
cosh2 u sinh2 u cos2 v sin2 v
[PTU, 2009]

Solution: We have x + iy = cosh(u + iv) = cos i(u + iv)


= cos iu cos v + sin iu sin v = cosh u cos v + i sinh u sin v
∴ Separating real and imaginary parts, we get
x = cosh u cos v; y = sinh u sin v

x y
i.e. = cos v and = sin v
cosh u sinh u
Squaring and adding, we get the first result.
x y
Again = cosh v and = sinh u
cos v sin v
Squaring and subtracting, we get the second result.

α  1
θ + iϕ
Example 57: If tan(θ ϕ) = tan α + i sec α, prove that e2ϕ = ± cot and 2θ = n + π + α.
2  2
[NIT Kurukshetra, 2005; KUK, 2009]

Solution: tan(θ + iϕ) = tan α + i sec α = sin α + i 1 = sin α + i …(1)


cos α cos α cos α
sin α 1 sin α − i
tan(θ − iϕ) = tan α − i sec α = −i = … (2)
cos α cos α cos α
We can write equations (1) and (2) in the form as
θ + iϕ = tan–1(tan α + i sec α) … (3)
θ – iϕ = tan–1(tan α
– i sec θ) … (4)
Adding (3) and (4), we have
2θ = tan–1(tan α + i sec α) + tan–1(tan θ – i sec α)

 2 tan α 
= tan −1  
 1 − (tan α + sec α) 
2 2

2 tan α 
= tan−1 
 − 2 tan2 α 

= tan−1  −
1 
 tan α 
= tan−1 [ − cot α ]
 π 
= tan −1  tan  + α 
 2 
Complex Numbers and Functions 475

 π 
= tan−1  tan  nπ + + α  =  n +  π + α
1
  2    2

2θ =  n +  π + α
1
 2
Similarly subtracting (4) from (3), we have
iϕ = tan–1(tan α + i sec α)– tan–1(tan α – i sec α)
 2i sec α 
= tan −1  
 1 + (tan2
α + sec2
α ) 
 2i sec α 
= tan−1  
 2(1 + tan α) 
2

sec α sec α
tan(2iϕ) = i =i
1 + tan2 α sin2 α
1+
cos2 α
i tanh 2ϕ = i sec α cos α, because sin2α + cos2α = 1
2

e2ϕ − e−2ϕ
tanh 2ϕ = = cos α
e2ϕ + e−2ϕ
By componendo and dividendo

α
2 cos2
1 + cos α 2 = cot2 α implying e2ϕ = ± cot α
e = 4ϕ
=
1 − cos α 2 sin2 α 2 2
2
Problem 58: If x = 2 cos α cosh β, y = 2 sin α sinh β , prove that sec(α + iβ) + sec(α − iβ)
4x
= . [NIT Kurukshetra, 2003, 2007]
x2 + y2
Solution: LHS = sec(α + iβ) + sec(α – iβ).
1 1 cos(α + iβ) + cos(α − iβ)
= + =
cos(α + iβ) cos(α − iβ) cos(α + iβ)cos(α − iβ)

[2 cos α cos iβ]


=
(cos α cos iβ − sin α sin iβ)(cos α cos iβ + sin α sin iβ) '

CD Formula C+D C−D


2 cos cos
2 2
2 cos α cosh β
=
[(cos α cosh β − i sin α sinh β)(cos α cosh β + i sin α sinh β)]
x 4x
= = 2
 x − i y   x + i y  x + y2
  
2 2  2 2
476 Engineering Mathematics through Applications

Example 59: Reduce tan –1 (cos θ + i sin θ ) to the from (a + ib). Hence show that
nπ π  i π θ
tan− 1 eiθ =  + − log tan  − 
 2 4  2  4 2
OR
If tan(a + ib) = cos θ + i sin θ , where letters denote the real quantities, prove that
nπ π  π + θ ,
a= + and e2b = tan where n is the integer. [PTU, 2007]
2 4  4 2

Solution: Given tan–1(cos θ + i sin θ) = a + ib


so that tan–1(cos θ – i sin θ) = a – ib
∴ 2a = tan–1(cos θ + i sin θ) + tan–1(cos θ – i sin θ) …(1)
 (cos θ + i sin θ) + (cos θ − i sin θ) 
= tan−1  
 1 − (cos θ + i sin θ)(cos θ − i sin θ) 
 2 cos θ 
= tan−1  
 1 − (cos θ + i sin θ)(cos θ − i sin θ) 
 2 cos θ 
= tan−1  
 1 − (cos θ − i sin θ) 
2 2 2

 2 cos θ 
= tan−1  
 1 − (cos θ + sin θ) 
2 2

= tan–1 ∞
π nπ π 
2a =  nπ +  or a =  + …(2)
 2  2 4
Similarily,
2ib = tan–1(cosθ + i sinθ) – tan–1(cosθ – i sinθ)
 (cos θ + i sin θ) − (cos θ − i sin θ) 
= tan−1  ,
 1 + (cos θ + i sin θ)(cos θ − i sin θ) 
 2i sin θ 
2ib = tan−1  2 
 1 + (cos θ + sin θ 
2

or tan 2ib = i sin θ implying i tanh 2b = i sin θ (³ tan ix = i tanh x)


tanh 2b = sin θ ...(3)
Taking inverse on both sides,
2b = tanh–1(sinθ)
1 + sin θ 
log 
1
2b =
2  1 − sin θ 

 cos2 θ + sin2 θ + 2 sin θ cos θ 


1  2 2 2 2
2b = log  
2 θ θ θ θ
 cos + sin − 2 sin cos 
2 2
 2 2 2 2
Complex Numbers and Functions 477

2
 cos θ + sin θ 
1  2 2
b = log 
4 θ θ
 cos − sin 
 2 2

2
 1 + tan θ 
1  2
⇒ b = log 
4 θ …(4)
 1 − tan 
 2

 π θ 
log  tan  +  
1
b= ...(5)
2   4 2 
Also from (4),
−2
 1 − tan θ 
1  2
b = log   (on interchange of numerator and denominator in (4))
4 θ
 1 + tan  ,
 2

−1  π θ 
i.e. b= log  tan  −   …(6)
2   4 2

 nπ + π  − i log tan  π − θ 
Hence, from (2) and (6), a + ib =
 2 4 2  4 2

π θ  π + θ
Also from (5), 2b = log tan  +  , i.e. e2b = tan
 4 2  4 2

Example 60: Separate cos–1(cos θ + i sin θ) into real and imaginary parts, where θ is a positive
acute angle.

Solution: Let cos–1(cos θ + i sin θ) = x + iy …(1)


∴ cos θ + i sin θ = cos(x + iy) = cos x cos iy – sin x sin iy
= cos x cosh y – i sin x sinh y
On comparing real and imaginary parts on both side,
cos θ = cos x cosh y
sin θ = − sin x sinh y } …(2)

Therefore, cos2θ + sin2θ = (cos x cosh y)2 + (– sin x sinh y)2


⇒ 1 = cos2x(1 + sinh2y) + sin2x sinh2y
⇒ 1 = cos2x + sinh2y(cos2x + sin2x)
⇒ 1 = cos2x + sinh2y
478 Engineering Mathematics through Applications

⇒ 1 – cos2x = sinh2y
sin2x = sinh2y …(3)
Now, sin θ = – sin x sinh y ⇒ sin2θ = sin2x sinh2y
⇒ sin2 θ = sin4 x [using(3)]
Since θ being given acute positive angle, sinθ is positive.
π
∴ sin2 x = sin θ = sin =1 …(4)
2
π
⇒ sin x = ± 1 ⇒ sin x = ± sin
2
π π π
⇒ x=± i.e. x lies between − and
2 2 2
Further, sin x = sin θ ⇒ x = sin−1 sin θ , (From (4))
Also, sin θ = –sin x sinh y, [from(2)]

⇒ sin θ = − sin θ sinh y

⇒ sinh y = − sin θ = α (say)

⇒ y = sinh−1 α = log α + α2 + 1

⇒ y = log − sin θ + sin θ + 1

β , prove that sin2α and cosh2β are the roots of the


Example 61: If sin –1(u + iv) = α + iβ
2 2 2 2
equation x + (1 + u + v )x + u = 0 [NIT Kurukshetra, 2008, 2009]

Solution: Given sin–1(u + iv) = α + iβ


Taking inverses, u + iv = sin(α + i β) = sin α cosh β + i sinh β cosα
On comparing real and imaginary parts, u = sin α cosh β and v = sinh β cos α
u2 = sin2α cosh2β = sin2α(1 + sinh2β) = sin2α + sin2α sinh2β …(1)
v2 = cos2α sinh2β = cos2α (cosh2β – 1) = cos2α cosh2β – cos2α …(2)
and 1+ u2 + v2 = sum of the roots
= 1 + [sin2α + sin2α sinh2β] + [cos2α cosh2β – cos2α]
= 1 + [(1 – cos2α) + sin2α (cosh2β – 1)] + [cos2α cosh2β – cos2α]
= 2 – 2 cos2α + sin2α cosh2β + cos2α cosh2β – sin2α
= [2(1 – cos2α) – sin2α] + cosh2β (sin2α + cos2α)
= sin2α + cosh2β

Alternately: x2 – (sum of the roots)x + product of the roots = 0


Complex Numbers and Functions 479

implies x2 – [1 + u2 + v2]x + u2 = 0
x2 + [1+ sin2α cosh2β + cos2α sinh2β]x – sin2α cosh2β = 0
x2 + [1 + sin2α cosh2β + cos2α(cosh2β – 1)]x – sin2α cosh2β = 0
x2 + [(1 – cos2α) + cosh2β(sin2α + cos2α)]x – sin2α cosh2β = 0
x2 + [sin2α + cosh2β)x – sin2α cosh2β = 0
(x – sin2α)(x – cosh2β) = 0
Implies x = sin2α, cosh2β are the roots of the above equation.

β, so that
Example 62: If cos–1(x + iy) = α + iβ
(i) x2 sec2α – y2 cosec2α = 1, (ii) x2 sech2 β + y2 cosech2 β = 1

Solution: We have, cos–1(x + iy) = α + iβ implies x + iy = cos(α + iβ) …(1)


and cos–1(x – iy) = α – iβ implies x – iy = cos(α – iβ) ...(2)
Adding (1) and (2), 2x = cos(α + iβ) + (α – iβ)
(α + iβ + α − iβ) (α + iβ − α + iβ)
= 2 cos cos = 2 cos α cos iβ
2 2
x = cos α cosh β …(3)
Subtracting equation (ii) from (i), similarly we get
2 iy = cos(α + iβ) – cos(α – iβ)
(α + iβ + α − iβ) (α − iβ − α − iβ)
= 2 sin sin = −2i sin α − sinh β
2 2
y = – sinα sinh β …(4)
From (3) and (4), we get
x2 y2
− = cosh2 β − sinh2 β = 1
cos α sin2 α
2

x2 y2
and + = sin2 α + cos2 α = 1
cosh2 β sinh2 β

π θ
Example 63: Prove that sin− 1(cosec θ) =  2nπ + (− 1)n  + i (− 1)n log cot
2 2

Solution: Let sin–1(csc θ) = z implies sin z = csc θ


eiz − e−iz
= csc θ or e2 iz – 2 i cscθ e iz – 1 = 0
2i

2i csc θ ± 4i2(csc θ)2 − 4  [ 2i csc θ ± 2i cot θ]


e = 
iz = = i [csc θ ± cot θ]
2 2
iz = log[i(csc θ ± cot θ)]
480 Engineering Mathematics through Applications


 1 + cos θ 
iz = log e 2 + log 
 sin θ 

π 1 1 + cos θ  π θ
z= + log   =  2n + (− 1)n  − i log cot 
2 i  sin θ  2  2

Alternately: Let sin–1(csc θ) = α + i β


Implying csc θ = sin(α + i β) = sin α cos i β + cos α sin i β
= sin α cosh β + i cos α sinh β …(1)
Equating real and imaginary parts,
csc θ = sin α cosh β …(i) 
0 = cos α sinh β …(ii) …(2)

π
From (ii), Either cos α = 0 implying α= …(3)
2
or sinh β = 0 implying β = 0
π
But β ≠ 0, since if β = 0 then cosh β = 1 and sin α = sin = 1, which is contrary to hold (i).
2
eβ + e−β
Substituting the value of α in (ii), csc θ = 1 ⋅ cosh β =
2
Implying e 2β – 2 csc θ eβ + 1 = 0, which is a quadratic in eβ and gives
θ
e2 β = csc θ + cot θ or β = log cot …(4)
2

ASSIGNMENT 7
1. If cos(α + i β) = r(cos θ + i sin θ), prove that
sin(α − θ) 1 sin(α − θ)
(i) e2β = (ii) β = log [Madras, 2003; KUK, 2005]
sin(α + θ) 2 sin(α + θ)
2. If cos(θ + i ϕ) = cos α + i sin α, pave that
(i) *sin2θ = ±sin α (ii)**cos 2θ + cosh2θ = 2 [Madras,**2000, *03; KUK, 2005, 2008]
3. If tan(A + iB) = x + iy,
(i) x2 + y2 + 2 x cot 2A = 1 (ii) x2 + y2 – 2y coth 2B + 1 = 0 (iii) xsinh 2B = y sin 2A

sin 2x tan u
4. If tan(x + iy) = sin(u + iv), prove that = .
sinh 2y tan v
 iπ 
5. If a + ib = tanh v + , prove that a2 + b2 =1.
 4
6. Prove that
Complex Numbers and Functions 481

(
(i) sin −1(ix) = 2nπ + i log x + 1 + x2 ) (ii) sin−1(cos ec θ) =
π
2
θ
+ i log cot .
2

 π + α
7. If tan(θ + i ϕ) = e iα, show that θ =  n + 1  π and ϕ = log tan
1
 2 2 2  4 2
8. Separate tan–1(x + iy) into real and imaginary parts.
9. Separate sin–1(cos θ + i sin θ) into real and imaginary parts, where θ is a positive acute
angle. [PTU, 2006; KUK, 2007]

6.8 SUM OF THE SERIES

Example 64: Find the sum of the series


e3α e5α
eα cos β − cos 3β + cos 5β − ……∞
3 5

e3α e5α
Solution: Let C = eα cos β − cos 3β + cos 5β − ……∞
3 5
e3α e5α
S = eα sin β − sin 3β + sin 5β − ……∞
3 5

e3α e5α
So that C + iS = eα (cos β + i sin β) − (cos 3β + i sin 3β) + (cos 5β + i sin 5β) − ……∞
3 5
e3(α+ iβ) e5(α+ iβ)
= eα+ iβ − + − ……∞
3 5
z3 z5
= z− + − ……∞, where z = eα + i β
3 5
= tan–1z = tan–1e(α + i β) …(1)
Similarly, C – iS = tan–1e(α – iβ) …(2)
Thus 2C = tan–1e(α + iβ) + tan–1e(α – iβ)

 eα+ iβ + eα − iβ   eα (eiβ + e−iβ ) 


= tan−1  = tan−1
 1 ( α iβ )( α −iβ ) 
 1 − eα+ iβ eα −iβ   − e e e e 
 eα ⋅ 2 cos β  2 cos β 
= tan−1   = tan−1  −α
 1− e 2α
  e − eα 
cos β
= tan−1  
 eα − e−α  
 −  
 2 
cos β 
= tan−1 
 − sinh α 
482 Engineering Mathematics through Applications

cos β 
tan−1  −
1 1
C= = − tan−1 cos β cosech α 
 sinh α 
Therefore,
2 2

Alternately: From(1),
C + iS = tan–1e(α + i β) = tan–1 eα(cos β + i sin β) = tan–1(eα cos β + i eα sinβ)

1 ,
+ i tanh−1 
2a 2b
is comparable to tan−1(a + ib) = 
2  1 − a2 − b2  1 + a2 + b2  

where a = eα cos β and b = eα sinβ.

1 2 1
Example 65: Sum the series x sin θ − ⋅ x sin 2θ + ⋅ x3 sin 3θ − … … …∞
2 3

1 2 1
Solution: Let S = x sin θ − x sin 2θ + ⋅ x3 sin 3θ − … … …∞
2 3
1 2 1
C = x cos θ − x cos 2θ + ⋅ x3 cos 3θ − … … …∞
2 3
Therefore,
x2 x3
C + iS = x(cos θ + i sin θ) − (cos 2θ + i sin 2θ) + (cos 3θ + i sin 3θ) − … …∞
2 3

x2 2iθ x3 3iθ
= xeiθ − e + e − … … …∞, (a logarithmic series)
2 3

z2 z3 z4
= z− + − + … … ∞ = log(1 + z), where z = xeiθ
2 3 4
= log[1 + x(cos θ + i sin θ) = log r cis θ

x sin θ
= log (1 + x cos θ)2 + (x sin θ)2  + i tan−1
1 + x cos θ

Equating imaginary parts, S = tan −1 x sin θ [except, when x cos θ = –1]


1 + x cos θ
Example 66: Find the sum of the series cos α + cos(α
α + β) + cos(α
α + 2 β)+ ………to n terms

Solution: Given C = cos α + cos(α + β) + cos(α + 2 β) + … to n terms


S = sin α + sin(α + β) + sin(α + 2 β) + … to n terms
Thus,
C + iS = eiα + ei(α + β) + ei(α + 2β) + … to n terms
= eiα(1 + eiβ + e2iβ + … to n terms)
Complex Numbers and Functions 483

 1(1 − einβ )   a(1 − rn ) 


= eiα  , Q sum of G.P. with n terms =
 1 − eiβ  1−r 

eiα (1 − e inβ )(1 − e−iβ )


=
(1 − e iβ )(1 − e−iβ )

eiα 1 − einβ − e−iβ + ei(n −1)β 


=
(1 − eiβ − e−iβ + 1)

 eiα − ei(α −β) − ei(α+nβ) + ei(α+ n −1)β 


C + iS =  
 2 − 2 cos β 
On taking real parts only,
cos α − cos(α − β) + cos(α + n − 1β) − cos(α + nβ) Q 1 − cos θ = 2 sin2 θ 
C= 
2.2 sin2
β 2
2

2 sin
α + (α − β)
sin
(α − β) − α
+ 2 sin 
(
 α + n − 1β + (α + nβ)  )  (α + nβ) − (α + n − 1β) 
 sin 
2 2  2   2 
=
β
4 sin2
2

2α − β  β 2n − 1β + 2α   β
−2 sin  sin   + 2 sin   sin  
 2   2   2 2
=

4 sin
2

 2n − 1β + 2α   2α − β 
sin 
  − sin 
2 2 
=
β (Using CD formula)
2 sin
2

2n − 1β + 2α 2α − β   2n − 1 + 2α − 2α − β 
2 cos  +  sin 
=
 2 2 2 2 
β
2 sin
2

(n − 1)  nβ
cos  α + β sin  
 2   2
C=
β
sin
2
484 Engineering Mathematics through Applications

1 2 1
Example 67: Find the sum of the series x sin θ − x sin 2θ + x3 sin 3θ − ………∞
2 3

1 2 1
Solution: Let S = x sin θ − x sin 2θ + x3 sin 3θ − ………∞
2 3
1 2 1
S = x cos θ − x cos 2θ + x3 cos 3θ − ………∞
2 3
1 2 1
∴ C + iS = x(cos θ + i sin θ) − x (cos 2θ + i sin 2θ) + x3 (cos 3θ + i sin 3θ) − ………∞ …(1)
2 3
1 2 2iθ 1 3 3iθ
= xeiθ − x e + x e − …………∞
2 3
z2 z3
=z− + − ……∞ (where z = x eiθ)
2 3
= log(1 + z) = log(1 + xei θ)
= log[(1 + x cosθ) + ix sin θ]
= log(r cis α) = log r + i α …(2)

 x sin θ 
where r = (1 + x cos θ )2 + ( x sin θ )2 and α = tan−1  
 1 + x cos θ 

x sin θ 
Whence α = tan−1   , except x = 1 and θ = (2 n + 1) π
 1 + x cos θ 

Example 68: Sum the series


1 + x cos α + x2 cos2α
α + x3cos 3α
α + ………… + xn–1 cos (n – 1) α
where x is less than unity. Also find the sum to infinity. [NIT Kurukshetra, 2007, 2008]

Solution: Let C = 1 + x cos α + x2 cos 2α + x3cos 3α + … + xn–1 cos (n – 1) α … (1)


S = x sin α + x2sin 2α + x3 sin 3α + … + xn–1 sin (n – 1) α …(2)
∴ C + iS = 1 + x(cos α + i sin α) + x2(cos 2 α + i sin 2 α) + … + xn–1 (cos n – 1 α + i sin n – 1 α)
= 1 + xeiα + x2e2iα + … + xn–1ei(n–1)α
= 1 + z + z2 + ……… + zn−1 , where z = x eiα
1(1 − zn ) 1 − xn eniα
= =
1− z 1 − xeiα
(1 − xn einα ) 1 − xe− iα
= ×
1 − xeiα 1 − xe− iα
1 − xn einα − xe− iα + xn+1 ei ( n −1 ) α
=
1 − x ( eiα + e− iα ) + x2 eiα e− iα
Complex Numbers and Functions 485

1 − xn (cos nα + i sin nα ) − x (cos α − i sin α ) + xn +1 [cos( n − 1) α + i sin( n − 1) α ]


∴ C + iS =
1 − 2 x cos α + x2
…(3)
On equating real and imaginary parts on both sides
1 − x cos α − xn cos nα + xn +1 cos( n − 1) α
∴ C= …(4)
1 − 2 x cos α + x2
Since it is given that x is numerically less than one.
⇒ xn, xn + 1 → 0 as n → ∞
1 − x (cos α − i sin α )
∴ The sum to infinity, C + iS = , (From (3))
1 − 2 x cos α + x2

1 − x cos α
and so that its real part is C=
1 − 2 x cos α + x2
Example 69: Find the sum of the series
sin α cos α + sin2 α cos2 α + sin3 α cos3α
α + …… + ∞

Solution: Let C = sin α cos α + sin2 α cos2 α + sin3 α cos3 α + …… + ∞


S = sin α sin α + sin2 α sin2 α + sin3α sin3α + …… + ∞
∴ C + iS = sin α eiα + sin2 α e2iα + sin3 α e3iα + …… + ∞
= z + z2 + z3 + … + ∞, where z = sin α eiα
z sin α . eiα
= =
1 − z 1 − sin α . eiα
sin α . eiα 1 − sin α . e− iα
= ×
1 − sin α . eiα 1 − sin α . e− iα
sin α ( eiα − sin α )
=
1 − sin α .( eiα + e− iα ) + sin2 α eiα e− iα

sin α (cos α + i sin α − sin α)


=
1 − 2 sin α cos α + sin2 α

sin α (cos α − sin α) + i sin2 α


C + iS =
1 − sin 2α + sin2 α
sin α (cos α − sin α)
∴ C=
1 − sin 2α + sin2 α
c3 c5
Example 70: Find the sum of the series c sin α + sin 3α + sin5α + ………∞
3 5
c3 c5
Solution: Let S = c sin α + sin 3α + sin 5α + ………∞
3 5
486 Engineering Mathematics through Applications

c3 c5
C = c cos α + cos 3α + cos 5α + ………∞
3 5

c3 c5
∴ C + iS = c(cos α + i sin α) + (cos 3α + i sin 3α) + (cos 5α + i sin 5α) + ………∞
3 5

c3 3iα c5 5iα
= ceiα + e + e + ………∞
3 5
= tanh–1(ceiα) …(1)
Likewise,
C – iS = tan h–1(ce–i α) …(2)
Subtracting (2) from (1),
 −1 −1 −1  A − B  
2iS = tanh−1(ceiα ) − tanh−1(ce−iα ) ,  using tanh A − tanh B = tanh  1 − AB  
 

ceiα − ce−iα 2ic sin α  2c sin α 


= tanh−1 = tanh−1 = tanh−1 i 
1 − ceiα ce−iα 1 − c2  1 − c2 

2c sin α 
2iS = i tan−1 
 1 − c2 

2c sin α 
tan−1 
1
or S=  …(3)
2  1 − c2 
 A+B 
Note: For finding C, take sum (1) and (2) and use tan−1 A + tanh−1 B = tanh−1 
 1 + AB 

Alternately: C + iS = tanh−1(ceiα ) = 1 log 1 + ce and proceed further for simplification and

2 1 − ce
separation of real and imaginary parts.

sin2 θ
Example 71: Sum the series cos θ + sin θ cos 2θ + cos 3θ + ………∞
1.2

sin2 θ
Solution: Let C = cos θ + sin θ cos 2θ + cos 3θ + ………∞
1.2
sin2 θ
S = sin θ + sin θ sin 2θ + sin 3θ + ………∞
1.2
sin2 θ 3iθ
∴ C + iS = eiθ + sin θ e2iθ + e + ………∞
1.2
 sin θ eiθ sin2 θ e2iθ 
= eiθ 1 + + + ………∞
 1 1.2 
 z z2 z3 
= eiθ 1 + + + + ………∞ 
 1 2! 3! 
Complex Numbers and Functions 487

= e i θ(e z) [where z = sin θ ei θ = sin θ (cos θ + i sin θ)]


= ei θ[e sin θcosθ + i sin θ]
2

= (e i θe i sin θ) × e sinθ cos θ


2

= esin θcos θ ei(θ+sin 


2 θ)

= e sin θ cos θ[cos(θ + sin 2 θ) + i sin(θ + sin 2 θ)]


∴ C = esin θ cos θ  cos(θ + sin2 θ) and S = esin θ cos θ  sin(θ + sin2 θ)

Example 72: Find the sum of the series


sin(α + 2β) sin(α + 4β)
sin α − + − ………∞
2! 4!
cos(α + 2β) cos(α + 4β)
and cos α − + − ………∞
2! 4!

sin(α + 2β) sin(α + 4β)


Solution: Let S = sin α − + − ………∞
2! 4!
cos(α + 2β) cos(α + 4β)
C = cos α − + − ………∞
2! 4!

ei(α+ 2β) ei(α+ 4β)


∴ C + iS = eiα − + − ………∞
2! 4!

 e2iβ e4iβ 
= eiα 1 − + − ………∞ 
 2! 4! 

 z2 z4 
= eiα 1 − + − ………∞  , where z = eiβ
 2! 4! 

= eiα [cos z ] = eiα  cos(eiβ )

= ei α[cos (cos β + i sin β)]


= ei α[cos (cos β) cos (i sin β) – sin (cos β)sin(i sin β)]
= ei α[cos (cos β)cosh(sin β) – i sin(cos β) sinh (sin β)]
C + iS = (cos α + i sin α)[cos A cos hB – i sin A sinh B]

Example 73: Find the sum to of the infinite series


1 1.3 1.3.5
1− cos θ + cos 2θ − cos 3θ + ……… (−π < θ < π)
2 2.4 2.4.6
1 1.3 1.3.5
Solution: Let C = 1 − cos θ + cos 2θ − cos 3θ + ………∞
2 2.4 2.4.6
488 Engineering Mathematics through Applications

1 1.3 1.3.5
and S=0− sin θ + sin 2θ − sin 3θ + ………∞
2 2.4 2.4.6
1 iθ 1.3 2iθ 1.3.5 3iθ
∴ C + iS = 1 − e + e − e − ………
2 2.4 2.4.6

−  − − 1 −  − − 1  − − 2
1 1 1 1 1
      3iθ
= 1 +  −  eiθ +
1 2 2 e2iθ + 2 2 2 e + ………
 2 1.2 1.2.3
= (1 + eiθ )−1/2 = (1 + cos θ + i sin θ)−1/2
1

θ θ θ
=  2 cos2 + i.2sin cos 
2
 2 2 2
− 1/2 − 1/2
θ  cos θ + i sin θ 
=  2 cos 
 2  2 2
− 1/2
θ  cos θ − i sin θ 
=  2 cos 
 2  4 4
− 1/2
Equating the real parts, we have C =  2 cos θ  θ
cos .
 2 4

ASSIGNMENT 8
Sum the following series:
1 1
1. cos θ − cos 2θ + cos 3θ − ………∞ [SVTU, 2006]
2 3
x2 x3
2. x cos θ − cos 2θ + cos 3θ − ………∞
2 3
n(n + 1) n(n + 1)(n + 2)
3. n sin α + sin 2α + sin 3α + ………∞ [NIT Kurukshetra, 2009]
1.2 1.2.3
1 1 1
4. sin θ − sin 2θ sin2 θ + sin 3θ sin3 θ − sin 4θ sin4 θ + ………∞
2
[SVTU, 2005; 2006]
2 3 4
x2 x3
5. cos α + x cos(α + β) + cos(α + 2β) + cos(α + 3β) + ………∞
2! 3!
 Hint : C + iS = eiα ez , where z = xeiβ 

x2 x3
6. sin α + x sin(α + β) + sin(α + 2β) + sin(α + 3β) + ………∞ [PTU, 2008]
2! 3!
 Hint : C + iS = eiα ez , z = xeiβ 
Complex Numbers and Functions 489

7. cos α sin α + cos2 α sin 2α + ……… ∞


[Hint: G.P. with common ratio: cos α ei α]
1 1.3 1.3.5
8. 1 + cos 2α − cos 4α + cos 6α − ………∞
2 2.4 2.4.6

1 1 2
9. 1 + cos x + cos 2x + cos 3x + ………∞
3 9 27

10. sin α + sin(α + 2β) + ……… + sin(α + n − 1)β [PTU, 2009]


[Hint: G.P. with common ratio: eiβ]

ANSWERS

Assignment 1

1 8i
(2 − 3) − i(2 + 3) (ii) −
2
1. (i)
25
α π−α π − α −
y
2. 2 sin cos + i sin 3.

2 2 2  x + y − 2x + 1
2 2

4. x = ±1.5 y = ±2 5. A circle, centre (– 1, 1), radius 2 units


1 1
6. (i) 2i (ii) − i
2 2

Assignment 2

1. Other vertices are 2i, − 3 + i, − 3 − i, − 2i, 3 − i


2. (i) P(z) represents a circle with centre A(a) and radius ‘k’.
(ii) P(z) is a straight line through A(a) making an angle α with OX.
3. (i) Annular region between the circles of radii 2 and 4 with centre (–3, 0) including
boundary of inner circle.
(ii) Region of complex plane about the line y = 2.

(iii) Infinite region bounded by the line θ = π and θ = π .


3 2
(iv) Region between the lines x = ±2 above the real axis.
4. Ellipse with foci at z = ±1 and major axis = 3.
6. (i) Right bisector of the line joining z = 3 and z = –1.
(ii) Circle through the points z = 3 and z = –1.
7. Locus of P(z) is a circle (unless k = 1, when locus is the right bisector of z – a and z – b)
490 Engineering Mathematics through Applications

Assignment 3

4mπ
3. n(n + 1)

Assignment 4
1 1
2. (i) 28 [0.98 ± i (0.195], 22 [−0.195 ± i(0.98)]

1
4n + 3
(ii) 25 cos , where n = 0, 1, 2, 3, 4
10

 π π  3π 3π
4.  cos ± i sin  ,  cos ± i sin 
5 5 5 5

 2nπ 2nπ 
5.  cos + i sin , n = 1, 2,… , 6
7 7 

(−1 + i) (1 − i)
6. , 7. x3 – x2 – 2x + 1 = 0
√2 √2

Assignment 5
1. 32 cos 5 θ – 24 cos3 θ + 6 cos θ
10. – (2)11 (sin 12 θ – 2 sin 10θ – 4 sin 8 θ + 10 sin 6 θ + 5 sin 4 θ – 20 sin2 θ)
11. sin5 θ = A sin θ – B sin3 θ + C sin 5 θ

Assignment 6

4  π π
1. (i) loge 10 + i tan−1  ± 2nπ (ii) log1 + i (2n + 1) π (iii) e−(2n +1) 2 e− 2
3 
 2nπ+ π  π2
π
,  2n +  π − log 2
  1 −
3. (i) 2e 4 (ii) e 8 , loge 2
 4 4
a2 − b2 π −  π
5. 9. (i) Z = nπ + (−1)n 4i (ii) Z = i  2nπ + 
a2 + b2 2   2

Assignment 7

1 2x
1. α = 2 tan 1 − x2 − y2
−1
9. x = cos− 1 sin θ

1 2y
β= tanh−1 y = log  sin θ + 1 + sin2 θ 
2 1 + x 2 + y2
Complex Numbers and Functions 491

Assignment 8

 θ 1 n(π − α)  2 sin α 
n
1. log  2 cos 
2
2.
2
log(1 + 2x cos θ + x2 ) 3. sin
2 /  2

sin2 θ
4. S = tan
−1
5. C = ex cosβ[cos(α + x sin β)]
1 + cos θ sin θ

6. S = e x cosβ [sin(α + x sin β)] 7. S = cot α 8. C = [cos α (1 + cos α)]

 9 − 3 cos x 
9. 
 10 − 6 cos x 
10. {
S = sin α + (n − 1)
β
2} nβ
sin csc
2
β
2

You might also like